Categories
Exam Questions Johns Hopkins Macroeconomics

John Hopkins. Final exam for graduate macroeconomic theory. Aschheim, Christ, Mills. 1962

 

The only remarkable thing to note about the following macroeconomics examination from Johns Hopkins is its somewhat confusing scheme for allowing students to select from the questions. No heroic leaps of imagination were demanded of the examinees, which is humane I guess. But an artifact is an artifact, so duly transcribed, posted, and added to the collection.

______________________________

MACROECONOMIC THEORY 18.604
Final Examination, May 21, 1962

Messrs. [Joseph] Aschheim,
[Carl] Christ, and [Edwin] Mills

Answer all questions except:

either       (a) three of the 12-point questions in Part II.
or             (b) one of the 36-point questions in Part I.

Time: 3 hours (i.e., 180 minutes); total credit 180 points.

PART I. 36 points each.
  1. Compare the roles assigned to technological progress in major writings of Schumpeter and Solow.
  2. Write a short critical essay comparing either
    1. The growth models of Harrod and Domar, or
    2. The models of growth and fluctuations presented by Tobin (JPE 1955) and Duesenberry (Business Cycles and Economic Growth)
  3. Analyze the essential differences between the modern conventional theory of public debt and the recent reformulation of this theory.
  4. The stability of equilibrium in the Wicksellian monetary system has been subjected to opposing interpretations by Myrdal and Patinkin. Review these opposing interpretations in light of Wicksell’s own formulation.
PART II. 12 points each.
  1. Saving equals investment.
  2. The demand for money (as a stock) depends on bondholdings as well as on income and interest rates.
  3. Disarmament would create a major depression in the United States.
  4. The effect of an increase in government expenditure does not depend on how the extra expenditure is financed, as long as it does not come from increased taxes.
  5. If national income is $500 billion and consumption is $400 billion, then for each increase of $1 in government expenditure the equilibrium level of national income will increase by $5.
  6. The multiplier analysis is useful for studying economic growth, abstracting from cyclical fluctuations.

Source: Johns Hopkins University, Sheridan Libraries, Ferdinand Hamburger University Archives. Department of Political Economy, Box 3/1 Series 6 , Folder “Graduate Exams 1933-1965” (sic).

Source: Professor Carl Christ in the Johns Hopkins University yearbook, Hullabaloo 1964, p. 42.

Categories
Economics Programs Johns Hopkins

Johns Hopkins. Ten-Year Projects and Outlook for Department. 1968-1978

The following ten-year plan (1968-1978) proposal for the department of political economy of Johns Hopkins was most likely drafted by senior members of the department, though the precise author(s) is not clear from the document itself. The bottom line of this plan is a request to be allowed to expand the deparment’s faculty and graduate student body by by half and by two-thirds, respectively. Otherwise the department feared  the loss of its national reputation due to having a reduced scope and scale.

The plan is at least as interesting for its obiter dicta regarding e.g., air-conditioning, computer terminals, secretarial staff, etc. 

_______________________

TEN-YEAR PROJECTS AND OUTLOOK FOR
DEPARTMENT OF POLITICAL ECONOMY
[Draft, 1967]

I. Introduction and Summary

The Department of Political Economy, like the rest of the University, has concentrated on small numbers and high quality in its research and instructional programs. It is our intention to continue that tradition.
During the early postwar period, this Department produced as large a group of outstanding young economists as almost any university in the country. Its small faculty included several of the country’s most eminent economists. Between 1958 and 1961 the Department was plagued with resignations of senior faculty. At the same time resources available at other universities were growing rapidly. As a result the Department lost its former status as a major producer of research and scholars. Since 1961, the Department has steadily been rebuilt and is again able to attract its share of outstanding faculty and graduate students. But the Department still suffers to some extent from the factors responsible for its earlier troubles: its small size and limited resource base.
The Department now consists of 11 faculty and 45 resident Ph.D. candidates.
The program outlined in subsequent sections is designed to strengthen the Department by increasing Its size and financial base, while still permitting it to reap the advantages of its relatively small size.
During the next decade, the Department should grow to about 18 faculty members, or about 50%. Its Ph.D. candidates should grow to about 75, or by about two-thirds. Such growth is essential to add stability to the research and instructional programs, and to permit us to cover the growing number of specialties in the subject.
Growth will be expensive. Faculty salaries and graduate fellowships will continue to rise. And no university can retain excellence, let alone improve its position, without substantial budgetary increases. Within a decade, the Department’s budget for salaries and fellowships should almost triple.

II. Immediate Needs and Plans

A. In 1967-68, the Department has ten full-time faculty members, one joint appointment with Operations Research, and one faculty member whose major appointment is in the School of Public Health. (A second joint appointment with Public Health was made in Spring 1967, but the appointee will be in Pakistan for two years.) We have two vacancies. One is a professorship, and results from the Department’s having been permitted to replace Professor Evans prior to his retirement. We have appointed a visiting Professor to this post for 1968-69. The other vacant post is an Assistant Professorship, created in the spring of 1967.
The Department’s full-time faculty ought to expand to about 18 during the next decade. Three purposes would be served by such an expansion. First, it would provide the Department with more depth in the central specialties of economic theory and quantitative methods, so that a resignation or leave of absence would not disrupt the instructional program. Second, it would permit us to make appointments in important specialties not adequately covered by existing faculty. The inevitable increase in specialization through time makes gradual expansion necessary. Third, it would enable us to discharge our obligations to the instructional programs in international relations more adequately.
The Department is now actively seeking funds for the establishment of a Center for International Economic Studies within the Department. This Center would provide a focus for graduate instruction and research in the areas of international trade and economic development. The Center would provide a major substantive focus for the Department in addition to its present focus on economic theory and quantitative methods. In addition, it would help to fill a pressing social need, since the development of poor countries is perhaps the most pressing social problem of our time. Finally, it would permit us to exploit the unique advantages of the University’s proximity to Washington.
Although we are now strong in international economics, we are weak in economic development. Hence, a specialist in economic development is our most pressing need in terms of our proposed Center, our own graduate program, and our participation in the international relations programs. Our next highest priority is in the area of industrial organization, in which we now offer only one course in alternate years. Other fields in which we need additional strength are economic growth, public finance, private finance, econometrics, managerial economics, and Soviet-type economics.
Our needs are not equally urgent in all these areas. And not every specialty requires a separate appointment. Individual scholars often have interests in two or more specialties. Finally, the importance of particular specialties, and the interests of individual faculty members change gradually through time.

B. The most important research facilities for the Economics Department are the library and computational facilities. In both cases, the special needs of the Department will make it increasingly important in the coming decade to supplement the facilities available to the University as a whole.
For many years the Department has felt the need for a workroom where copies of major journals and reference books could be kept. In an important sense, the technical journals and data sources play the role in economics that the laboratory plays in an experimental. science. The movement of the library from Gilman Hall has imposed a major burden on faculty in the Department. In addition, faculty and graduate students in economics are now sufficiently numerous that duplicate copies of major journals are essential. We have made a small beginning toward meeting our library needs by establishing a workroom in our new quarters. A very limited number of journals is being purchased from research funds. In the coming years it will be important to expand the number of journals in our workroom, and to add major reference and data volumes. If a new social science building is constructed, or if the Department is able to expand its quarters as a result of the construction of a humanities building, a departmental library should be a major planning item.
The Department now has 6 desk calculators for use by faculty and students. Most are old and should be replaced with more modern machines within a few years. In the next decade we should at least double the number of calculators available. Some of our faculty now make frequent use of one or more of the real time-saving consoles located around the University. Within a short time, it will be important for the Department to have one or more such consoles in or near the Department area.
The Department now has two full-time and one half-time secretary. The half-time secretary is financed from research funds. Within a year or two she will need to be full time. Within a decade we will probably need five full-time secretaries. We need one additional electric typewriter this fall, and at least three modern tape recorders. During the coming years wo will need several additional typewriters and recorders, and other minor items of office equipment.

C. In the spring of 1967, we substantially revised both our undergraduate and graduate curricula. At the undergraduate level, the major change was to permit most courses to be taken after only two semesters economic theory rather than three as was previously required. This opened up, several courses in the Department to international relations majors and others outside the economics major. At the graduate level, the major change was to provide a more concentrated and integrated program in economic theory for first-year Ph.D. candidates.
During the last few years, the number of undergraduate registrations in economics courses has grown much more rapidly than the undergraduate student body. This is shown in the following table of selected registrations.

1963-64 1964-65 1965-66 1966-67
18.1 241 339 351 358
18.2 50 85 121 107
18.3 50 79 94 108
18.301-302 51 56 52 74
Total 392 559 618 647

This has necessitated our giving some courses each semester which were previously given in alternate semesters. Presumably, future growth in undergraduate registrations will more nearly approximate the growth in the student body. During the next few years our major need at the undergraduate level is to add a few specialized courses that will be available to students with a limited background in economics. Planned economics and urban economics are examples of such courses.
Our Ph.D. program is now too small. We do not have enough students to justify graduate courses in specialties which should be covered in a high quality graduate program, and we do not have enough faculty to offer the courses. We thus need to expand the graduate enrollment and the faculty simultaneously in order to be able to fill gaps in our graduate program in areas such as economic development, fiscal policy and industrial organization.

D. This Department is far smaller than any other major graduate department in economics. The next smallest, Princeton, is approximately the size that our projections indicate we will be in 10 years. Others are much larger.
We do not aspire to match the size and growth of most of the departments with which we compete for faculty and graduate students. We are firmly convinced of the advantages of smallness. But until very recently our size was almost below that required for viability. And we see clear advantages in some further growth, which would still retain the benefits of our relatively small size.

III. In this section I will discuss the undergraduate and graduate instructional programs, and faculty research activities in that order.

Undergraduates can either concentrate or major in economics. Although there is some tendency for better students to major rather than concentrate, some very able students choose the less intensive program. A stronger tendency is for those whose goal is a Ph.D. program in economics to major, and for others to concentrate.
An average senior class contains about 15 concentrators in economics. Some of these graduates take jobs, but many go to graduate school in business, law and economics.
An average senior class contains about 10 majors in economics. Although a few majors take jobs upon graduation, most attend graduate school in economics or business. And the program is designed with this group in mind. In recent years, our majors have undertaken successful graduate study at Chicago, Stanford, M.I.T., Johns Hopkins and other leading institutions. The Department’s requirements of a major include four semesters of economic theory, economic history, a year of statistics, a year of mathematics, a senior essay, and work on one or more advanced fields. We feel that our majors are as well prepared for graduate study as those at any university in the country.
For many years, the goal of our Ph.D. program has been to provide thorough training in economic theory, quantitative methods, and a small number of substantive fields to a small group of high quality students, most of whom intend to enter teaching and research posts. In the years 1950-1966, 63 people received the Ph.D. for work in this Department. This comes to 3.7 per year, but there is a slight upward trend, and we have given about five per year in recent years. Among them are some of the leading academic economists of the postwar generation. Our graduates hold posts at Yale, Chicago, Minnesota, Northwestern, Purdue, Wisconsin and other leading United States universities. They also hold major academic posts in the U.K., Israel, Japan and Australia.
In the early postwar period, Johns Hopkins had among its Ph.D. candidates more than its share of the best students who studied economics. This resulted from the high quality of the faculty, the small and personal nature of the Ph.D. program, and the ability of the Department to offer fellowships that were larger than those offered by competing institutions. In the late 1950’s, this situation changed, partly because of the loss of most of the Department’s senior faculty. Since 1981, the Department has been substantially rebuilt; and is again among the leading economics departments in this country. We have greatly improved the quality of the student body, and are now getting about our share of the best graduate students, but we have not regained our former edge. To do so is the goal of the plans outlined in other sections of this report.
In the Political Economy Department, as elsewhere in the University, most faculty research is basic rather than applied. Within that framework, however, a wide spectrum of subjects and techniques is encompassed. Some of the research is purely theoretical, employing mathematical and logical tools to improve our understanding of economic phenomenon. Most of the research, however, is quantitative, employing not only economic theory but also statistical methods and data.

IV. Relationship to the Hopkins community

A. At the undergraduate level, the enrollment in economics courses has grown rapidly in recent years. Nearly every undergraduate now takes at least one economics course. And for several years we have had more than a hundred students per year in each of our second and third courses in economic theory. About 50 students per year enroll in our course in current economic problems. In 1967-68, the Department will offer 11 semester courses at the 0-99 level, and 13 semester courses at the 300-level, all of them open to undergraduates who are not economics majors.
At the graduate level our Ph.D. candidates frequently take courses in the Departments of Mathematics, Statistics, and Operations Research. Less frequently they take courses in the Departments of Political Science, History, Geography, and Social Relations. Frequently, 300- and 600-level courses in economics are taken by Ph.D. candidates in Operations Research, Environmental Sciences, Statistics, and Geography. Less frequently, they are taken by students in History, Social Relations and Political Science. Sometimes, students from SAIS take our courses in international economics and economic development.
In recent years, there has been a considerable increase in the exchange of graduate students between this Department and others for course work. We expect this trend to continue and feel that it should be encouraged.

B. The Center for International Studies will be established within the Department of Political Economy. However, many problems within the Center’s purview require interdisciplinary study, and we hope to use the Center as a vehicle for joint teaching and research programs. SAIS is the most natural partner for such ventures, but we hope to explore possibilities with Homewood departments also.

C. The Department takes an active part in the A.B.-M.A. and Ph.D. programs in international relations. We give year courses in international economics and economic development mainly for students in these programs. In our curriculum revision last spring, we reduced the prerequisites for these courses to make them more accessible to international relations specialists. We are generally pleased with our success in staffing the economics part of the international relations program. However, we feel a need for a major appointment in economic development before we can be fully satisfied with our contribution.
The Political Economy Department has one joint appointment with the Operations Research Department. In addition, wo have two faculty members in the Department whose major appointments are in the School of Public Health. The Department has no fixed policy regarding joint appointments. Those that wo have are successful because of special circumstances in which such an arrangement is in the interests of all parties. We expect that such circumstances will arise again. But we think it unwise to plan for certain numbers or kinds of joint appointments.

V. Instructional Program

A. The following table summarizes the Department’s instructional program in 1967-68:

Course Number No. of Courses Hours Per Week Credit Hours No. of Courses Hours Per Week Credit Hours No. of Courses Hours Per Week Credit Hours
0 – 99 5 14 14 6 17 17 11 31 31
300-399 7 15 22 ½ 6 13 19 ½ 13 23 42
600-699 11 23 12 24 23 47
Total 23 52 24 54 47 106

Each full-time faculty member except the chairman teaches two courses per semester. The chairman teaches three courses per year. All faculty attend the weekly Department seminar. Most faculty members will attend our dissertation seminar several times a year.
All courses numbered 0-99 are open to all qualified undergraduates, whether they are economics majors or not. All 300-399 numbered courses are open to qualified undergraduates and to graduate students from other departments. A few are not normally taken by Ph.D. candidates in economics. 600-699 numbered courses are open to graduate students in this and other departments.
It is difficult to predict future growth of undergraduate enrollment since, as stated above, we expect it to grow about as fast as the undergraduate student body, which we do not control. However, even in the absence of substantial growth in enrollments, there are several courses that should be added either at the 0-99 or the 300-399 level. These include comparative economic systems, corporation finance, public finance, and economic growth. Some other courses, now given only in alternate years, should be given every year. These include industrial organization, economics of education, and urban economics. Substantial growth in enrollments would require that we offer additional sections of some courses and that we offer some courses every semester rather than once a year.
At the graduate level, our intake of students has been between 10 and 15 for several years, resulting in a body of about 35 students in residence. We have now embarked on a conscious program of increasing the size of our graduate program; in 1967-68, 18 students entered and our student body is 45. Our intake should increase gradually over the coming decade to about 25, with a resulting student body of about 75. Seventy-five is the present graduate enrollment of the next smallest of major graduate programs in economics in other universities. Others are considerably larger. We feel that this growth is necessary to enable us to offer the range of courses now required for proper coverage of our subject matter.
Unless a major expansion of the international relations program is undertaken, we should not have to devote more faculty resources to it, once we have made the appointment we are now seeking in economic development.
Expansion of the faculty from 11 to 18 would permit the addition of 28 semester courses in the Department. The exact nature and level of the courses to added will depend on the interests of faculty members recruited, the interests of undergraduate and graduate students, and developments in the subject matter. However, we expect to continue the policy of devoting roughly half the Department’s teaching resources to courses numbered 0-99 and 300-399, and the other half to 600-level courses.

B. The Department completely reorganized both its undergraduate and graduate curricula in the spring of 1967. This reorganization permitted us to identify clearly the gaps in our program referred to in Section II. We feel that our only pressing curriculum need is now to fill these gaps. Major curriculum reform becomes necessary periodically in a developing discipline, but we have no plans for further reform.

VI. Resources Outside the University

The Department has no formal relationship with organizations outside the University. The Department does, however, benefit from proximity to Washington in several ways. First, proximity to Washington is an attraction to some actual and prospective faculty members. They may obtain data, attend meetings and seminars, and occasionally undertake paid consulting at U.S. Government agencies, international organizations, or private research Institutions. Second, Washington is an attractive source of summer jobs for our graduate students, and a few of our graduates take permanent posts there.

VII. Space requirements

In the spring of 1967 the Department moved into new quarters on the fourth floor of Gilman Hall. These quarters are an important improvement over those previously available to the Department. The new quarters consist of 12 faculty offices, a departmental office, a calculator room, 11 small cubicles for graduate students, a seminar room, and a workroom where recent technical journals are kept.
In terms of space needs, however, the now quarters are already inadequate. We now have 13 faculty posts in the Department, but only 12 offices. In fall 1968 we expect to have all 13 posts filled, and we will have the Hinkley Professor in the Department. We will thus be two offices short. In addition, we recently hired a part-time secretary. The Department office is adequate for only the two secretaries now occupying it and we have to house the new secretary in the calculator room. Within the next year the part-time post will have to be made full time, and the housing problem will be acute.
The ten-year projection for the Department will require major additions to the Department’s space facilities. Faculty offices will have to expand from 12 to 18. The secretarial force will have to expand to at least five, and that will require at least two rooms entirely devoted to secretarial use. The Department now has one seminar room. Virtually all our 300- and 600-level courses are held there and it is in use more than 35 hours per week within a short time it will be necessary to have an additional seminar room. Within ten years it will be important to have a third room that can be used for seminars, conferences and other meetings. Within the next few years we will need a larger calculator room. We already need additional calculators, and this need will grow as the faculty and graduate student body grows. In addition, we will shortly need one or more real time sharing consoles in the Department area.
It is clear that a building to house either the social or behavioral sciences is already overdue at Johns Hopkins. Despite all the building on the campus in the last decade, the social sciences and humanities – as well as statistics and various ancillary facilities are still all housed In Gilman Hall. It is virtually the only building on the campus that is not fully air conditioned. And the removal of the main library has worsened the situation.
The nature of this Department’s space needs would make it difficult, but not impossible, to satisfy them by regrouping the Gilman facilities if some other departments were to be housed in other buildings. A social or behavioral science building – which would include economics ought to be a major part of the 10-year fund raising program.

VIII. Tables and Graphs

A. The following table shows the undergraduate concentrators and majors in Political Economy for 1967-68:

Concentrators Majors
Juniors 5 10
Seniors 16 9

This table does not include the BIM students.
In 1967-68 the Department has 18 entering and 27 returning graduate students. We have no post-doctoral students.

B. Faculty

Edwin S. Mills – Professor and Chairman

Age: 39
econometrics, statistics, microeconomics
Research projects: [blank]

Bela Balassa – Professor

Age: 39
International trade, economic theory, comparative systems, economic development
Research projects: [blank]

Carl F. Christ – Professor

Age: 44
econometrics, macroeconomics, money
Research projects: [blank]

G. Heberton Evans, Jr. – Professor

Age: 67
economic history, history of economic thought, private finance
Research projects: [blank]

Herbert E. Klarman – Professor

Age: 51
economics of health, public finance
Research projects: [blank]

Peter Newman – Professor

Age: 39
economic theory, mathematical economics, economic development
Research projects: [blank]

Jürg Niehans – Professor

Age: 48
economic theory, money
Research Projects: [blank]

Frederick T. Sparrow – Associate Professor

Age: [blank]
operations research, microeconomic theory, managerial economics
Research projects: [blank]

William Oakland – Assistant Professor

Age: 28
public finance, money, economic theory
Research projects: [blank]

John Owen – Assistant Professor

Age: 35
labor, economic theory, education
Research Projects: [blank]

William Poole – Assistant Professor

Age: 30
money, macroeconomics, international trade
Research projects: [blank]

H. Louis Stettler, III – Assistant Professor

Age: 29
economic history, economic theory, statistics
Research projects: [blank]

C. As was stated above, the Department should grow from its present size of 12 faculty members to 18 during the next decade. We feel that the current division by rank — about half the faculty are professors — is about right. The following table shows a feasible growth pattern to meet the projected goal:

1967-68

1968-69 1969-70 1970-71 1971-72 1972-73 1973-74 1974-75 1975-76 1976-77

1977-78

Prof.

6 7 7 7 7 7 8 8 8 8 9
Assoc. Prof. 1 1 2 2 2 2 2 2 2 2

2

Asst. Prof.

4 5 5 5 6 6 6 6 7 7 7
Total 11 13 14 14 15 15 16 16 17 17

18

Our priorities among specialties were indicated in Section II. Beyond that, it is not possible to indicate which appointments should be made in which years and at which levels. Much depends on the availability of particular faculty in whom we are interested and on combinations of zfields in which prospective faculty are interested.

D. The Department is not persuaded that there is an important place for postdoctoral studies in economics during the next decade. Promising graduate students now obtain well-paid posts at universities with graduate programs and with relatively light teaching loads. Our impression is that it would be difficult to entice them to post-doctoral fellowships, and that there is little merit in doing so. Nor are we persuaded that there is a substantial group of young economists at small colleges who could produce significant books and papers if given a year off from heavy teaching duties. The only promising possibility seems to be to find a small number of young foreign scholars who have the Ph.D. or its equivalent, and who could spend a year here with mutual benefit to themselves and to us. The Department is not prepared to urge such a program at this time.

E. The accompanying table shows a projected ton-year budget for the Department of Political Economy. The personnel item includes base salaries and fringe benefits of faculty, secretaries and junior instructors. It assumes that faculty salaries will rise by 7% per year over the next decade. It also takes account of the faculty expansion projected in Section E.
The fellowship budget includes graduate fellowships, tuition and stipends, from whatever source. At present, some is University money, some is U.S. Government money funneled through the University (NDEA, NSF), some is fellowship money obtained by students with Department recommendations, and some is money obtained by students (mostly foreigners) entirely on their own (from foreign sources, U.S. State Department, foundations). This budget assumes that fellowships per student will rise by about 5% per year during the next decade. The table also assumes that the number of entering students will rise from 18 to 25, and the total graduate student body from 45 to 75, over the next decade.
The third line projects a growth of the Department’s incidental and telephone accounts by about 5% per year over the decade.
Excluded from the table are research funds for supplemental faculty salary, research assistants, or computing. No attempt has been made to project funds available from sponsored research or from University sources such as the faculty research grants fund.

1967-68

1968-69 1969-70 1970-71 1971-72

1972-73

Personnel

233,640 250,000 306,400 340,700 356,500 395,500
Fellowships 159,500 186,000 207,100 229,700 254,100

280,400

Telephones, Supplies

4,000 4,200 4,400 4,600 4,900 5,100
397,140 440,200 517,900 575,000 615,500

681,000

1973-74

1974-75 1975-76 1976-77 1977-78
Personnel 453,200 484,900 535,800 573,800

653,400

Fellowships

308,600 339,000 371,600 401,200 432,800
Telephones, Supplies 5,400 5,600 5,900 6,200

6,500

767,200

829,500 913,300 981,200

1,092,700

Source: Johns Hopkins University. The Eisenhower Library. Ferdinand Hamburger, Jr. Archives. Department of Political Economy [Records], Box 5, Folder “Planning Documents: 1938, 1965, 1967”.

Categories
Exam Questions Johns Hopkins Money and Banking

Johns Hopkins. Semester Exams for Monetary Economics. Musgrave, 1959-1960

 

From 1958 through 1962 Richard Musgrave was Professor of Economics at Johns Hopkins. One thinks of him today as a giant in the history of public finance but the examination below reminds us that he was also an economist who still taught graduate courses in monetary economics/policy at least into the early 1960s.

______________________

More about Richard Musgrave

All posts with the tag “Musgrave” here at Economics in the Rear-view Mirror.

In particular one post with biographical and career information.

______________________

Richard Musgrave
Faculty of Arts and Sciences — Memorial Minute

At a Meeting of the Faculty of Arts and Sciences April 8, 2008, the following Minute was placed upon the records.

Richard Musgrave, the Harold Hitchings Burbank Professor of Political Economy, Emeritus, was the leading public finance economist of his generation. He died on January 15, 2007, at the age of 96.

Richard Abel-Musgrave was born in Königstein, Germany, and educated in Munich and Heidelberg. He was of half Jewish ancestry, his paternal grandfather and maternal grandmother both being Jews who had converted to the Christian faith.

He came to the United States in 1933 as an exchange student at Rochester University but soon transferred to Harvard where he received his PhD in 1937. He decided not to return to Germany and applied for U.S. citizenship in that same year. At that time he dropped the hyphen in his family name, becoming Richard Abel Musgrave. He was known thereafter as Richard Musgrave.

After completing his PhD, Musgrave worked at the Board of Governors of the Federal Reserve until 1948. He then taught at Johns Hopkins, the University of Michigan and Princeton before joining the faculty at Harvard in 1965. He held simultaneous appointments in the economics department and in the Harvard Law School, the first person to hold a joint appointment in both the Faculty of Arts and Sciences and the Law School. Professor Musgrave took emeritus status in 1981 and moved to California where he was an adjunct professor at the University of California at Santa Cruz.

Although the 19th-century giants of political economy, David Ricardo and John Stuart Mill, wrote extensively about the theory of taxation, by the middle of the 20th century the teaching and writing on public finance in the United States was largely descriptive and institutional. Richard Musgrave changed all of that with his major volume, The Theory of Public Finance, published in 1959.

The Theory of Public Finance was both a theoretical research monograph and a text book. It applied the analytic tools of price theory and of Keynesian macroeconomics to the issues of tax incidence (i.e., who bears the burden of taxes), of efficiency (i.e., measuring the losses caused by the distorting effects of taxes), and of achieving full employment. All of this was done in a very readable and accessible way that made the book very widely studied. The book proved to be a particularly significant resource for tax law professors in their teaching and writing about federal tax policy.

A key feature of Musgrave’s Theory of Public Finance was the division of the problem of public finance into what Musgrave called three “branches.” One “branch” was devoted to the problem of achieving full employment. Here Musgrave applied the ideas of Keynesian fiscal policy to using tax reductions and government spending to increasing aggregate demand. A second “branch” focused on economic efficiency, i.e., on the design of taxes that would raise revenue with the least distortion to incentives and therefore the least loss of real incomes. The third “branch” then dealt with issues of redistribution to achieve a politically acceptable distribution of income. These branches were of course just pedagogical devices and not a way of organizing the actual making of policy.

Richard Musgrave was an inspiring teacher. It was clear to his students that he cared about both the analytic science in public finance and the practical implications of that analysis for improving our tax system. He taught students to think about the impact of taxes on economic efficiency while not losing sight of their distributional consequences. Or, as he might have said, to think about the distribution of the tax burden and the use of taxes and transfers to redistribute income while not losing sight of the consequences of the progressive tax and transfer structure on economic efficiency.

In the weekly graduate seminar in public finance, graduate students and visiting faculty would present their latest research. The seminar brought together not only graduate students and faculty from the department of economics, but also tax specialist members of the Harvard Law School faculty. Their presence added a greater degree of practical focus to the seminar’s discussion of tax reform. Musgrave’s questions and insights kept the seminar focused on the substantive importance of the problems rather than on the more abstract methodological issues. Many of the students taught by Richard Musgrave went on to do important work in public finance.

Although Musgrave felt strongly about tax policy and about transfer programs like Social Security and unemployment insurance, he was not an activist who tried to influence outcomes in Washington. He appeared to believe that he was most effective in developing the analysis and teaching students who would carry this material into practice.

An important exception to this was a major report on fiscal reform in Columbia that Musgrave prepared jointly with Malcolm Gillis in 1971. This report, prepared under the auspices of the Harvard International Tax Program of the Harvard Law School, was based on extensive and detailed work in Columbia.

Richard Musgrave was elected a Distinguished Fellow of the American Economic Association in 1978. Musgrave was one of the organizers of the International Seminar in Public Economics which brought together American and European faculty members who specialized in public finance. He also served as an honorary president of the International Institute of Public Finance.

Professor Musgrave collaborated with his wife, Peggy Musgrave, in writing a popular undergraduate text book, Public Finance in Theory and Practice, which was published in 1973. The Musgraves also found time to reach out to young colleagues and their wives at their homes in Belmont and in Vermont.

Respectfully submitted,

Lawrence Summers
Bernard Wolfman
Martin Feldstein, Chair

Source: The Harvard Gazette. June 12, 2008.

______________________

THE JOHNS HOPKINS UNIVERSITY

Economics 611
Final Examination
Prof. R. A. Musgrave
January 22, 1960

I

Write for forty-five minutes.

There is by now pretty general agreement, among monetary theorists, regarding the various relationships by which the supply of money may affect the level of output and prices. Nevertheless, there remains a division between those who prefer to study the role of money in the framework of an income-expenditure approach, and those who prefer the quantity theory of equation of exchange tradition. What, if any, substantive justification is there for retention of this dichotomy? If there is none, which approach is to be retained? If there is, what distinct purposes are served by the two approaches?

II

Write on two out of the following three questions, thirty minutes each,

  1. Various writers, including Wicksell, Fisher and Keynes, have treated the problem of monetary disequilibrium and the nature of the equilibrating process, in terms of the differential between two rates of interest. Discuss these approaches and compare the concepts of interest used therein.
  2. Where do you stand on the loanable funds—liquidity preference controversy? In particular, are you satisfied that the distinction between the stock and the flow approach to monetary theory is purely terminological?
  3. “It was a great misfortune for the development of monetary theory, that Marshall and Pigou did not stick with their initial intent to relate k to wealth, but proceeded to relate it to income. Thereby was postponed the recognition — so essential for a fruitful approach to monetary theory — that the demand for money must be dealt with in the context of a general portfolio theory.” Discuss.
III

Write on the following three statements, for fifteen minutes each. Indicate whether the statement is right or wrong and why.

  1. “The real balance effect implies that the demand schedule for money has unit elasticity, from which it follows that the price level changes proportionately with the money supply.”
  2. “The liquidity trap is a necessary but not a sufficient condition for under-employment equilibrium.”
  3. “Classical theory was mistaken in assuming that the rate of interest is determined by income independent of money supply. As Keynes has shown, interest is determined by money supply and then determines income.”

______________________

Dr. R. A. Musgrave
Friday, May 20, 1960

ECONOMICS 611
  1. The following changes occur: Bill holdings at the Federal Reserve rise by 100 million, while bond holdings fall by 80 million. Also, bank holdings of bills fall by 70 million, non-bank holdings of bills fall by 30 million, and non-bank holdings of bonds rise by 80 million. What is the resulting change in excess reserves, assuming a reserve ratio of 20%, and why? (Assume that the system retains such changes in excess reserves as result, without reacting with corresponding changes in loans.)
  2. Assume that the system is always loaned up. What will be the effects on member bank reserves and demand deposits of (a) an increase in vault cash by 100; (b) a decrease in currency in circulation by 200; (c) a gold outflow of 300; (d) a decrease in treasury deposits at commercial banks by 500. The reserve ratio is again 20%.

Source: Johns Hopkins University. The Eisenhower Library. Ferdinand Hamburger, Jr. Archives. Department of Political Economy [Records], Series 6/7, Box 3, Folder “Department of Political Economy, Graduate Exams 1933-1965”.

Image Source: Richard A. Musgrave page at the University of Michigan’s Faculty History Project.

Categories
Exam Questions Johns Hopkins Theory

Johns Hopkins. Exam for Welfare Economics. Lerner, 1958

 

Abba Lerner changed his academic locations (including leaves of absence to accept visiting positions) with a frequency rivaled by few. The academic year 1957-58 found him visiting the department of political economy at Johns Hopkins University. The artifact for this post is the final examination for Lerner’s course on welfare economics.

Lerner’s notes for seminars on social welfare functions held at the IMF and at the Cowles Commission in  1952.

_________________________

THE JOHNS HOPKINS UNIVERSITY
Welfare Economics
18-640

Final Examination May 20th 2-5 p.m., 1958.
Abba P. Lerner

Answer four questions, in separate blue books, in ink.

  1. Discuss the meaning, the validity and the significance of the proposition that it is impossible to derive a social welfare function from individual preference functions.
  2. How far can one carry the analogy between a political voting procedure and the economic price mechanism, and between the rationale of voting between alternative policies and that of allocating dollars between alternative purchases?
  3. Discuss the rational elements in relation to other elements in the social objectives of optimum distribution of income, optimum population, and optimum rate of saving.
  4. What is sound, what is unsound, and what is useful in the doctrine of consumers’ surplus?
  5. Why is it socially desirable to have the prices of products equal to the value of the marginal factors used in their production? How is this objective affected by equity elements such as the need for subsidies?
  6. Under what conditions would a partial freeing of trade be harmful to society in the largest sense? In your answer explain the treatment of this problem in terms of “second best” and the use of the concept of “divergence”.
  7. Compare the arguments for the imposition of trade restrictions for the sake of affecting the international terms of trade with those undertaken for the sake of affecting the domestic distribution of income. Give special attention to the interdependence of efficiency and equity considerations.

Source: Johns Hopkins University. The Eisenhower Library. Ferdinand Hamburger, Jr. Archives. Department of Political Economy [Records], Series 6/7, Box 3, Folder “Department of Political Economy, Graduate Exams 1933-1965”.

Image Source: Photograph of Abba Lerner printed in an announcement for his speech “Israel—The Next Ten Years” (February 25) at the 1958 Forum presented by Beth Emet the Free Synagogue (Evanston, Illinois). Library of Congress, Manuscript Division. The Papers of Abba P. Lerner, Box 6, Folder 8 “ ‘B’ miscellany”. A copy of the announcement was posted by Ellen Blum Barish in Tablet (January 14, 2014).

Categories
Business Cycles Distribution Economic History Exam Questions History of Economics Industrial Organization International Economics Johns Hopkins Labor Money and Banking Public Finance Public Utilities Statistics Theory

Johns Hopkins. General Written Exam for Economics PhD. 1956

 

One is struck by the relative weight of the history of economics in this four part (12 hours total) general examination for the PhD degree at Johns Hopkins in 1956. Also interesting to note just how many different areas are touched upon. Plenty of choice, but no place to hide.

________________________

Other General Exams from Johns Hopkins

________________________

GENERAL WRITTEN EXAMINATION FOR THE PH.D DEGREE
DEPARTMENT OF POLITICAL ECONOMY

*  *  *  *  *  *  *  *  *  *  *  *  *  *  *

PART I
June 4, 1956, 9-12 a.m.

Answer two questions, one from each group.

Group I.
  1. Write an essay on the theory of capital. It should include a discussion of the place of capital theory in economic analysis: for what purposes, if any, we need such a theory, Do not omit theories or issues which were important in the history of doctrines, even if you should regard them as irrelevant for modern analysis.
  2. Discuss and compare the capital theories of Böhm-Bawerk, Wicksell, and Hayek.
  3. Write an essay on the theory of income distribution. Organize it carefully, as if it were designed for an article in the Encyclopedia of the Social Sciences. Include discussions of alternative theories such as imputation theories, residual theories, surplus value theories, etc.
Group II.
  1. The following statements attempt to show that marginal productivity theory is inconsistent with factual observation. Accepting the stated facts as given, discuss whether they call for the rejection or major modification of the theory. If so, how? If not, why not?
    1. “In the most important industries in the United States wage rates are set by collective bargaining and are largely determined by the bargaining strength of the parties. Marginal productivity of labor is neither calculated nor mentioned in the process.”
    2. “In many industries competition among employers for workers is so limited that most firms are able to pay less than the marginal productivity of labor.”
    3. “Workers in some trades — say, carpenters or bricklayers — work essentially the same way as their predecessors did fifty years ago; yet their real wages have increased greatly, probably not less than in occupations where productivity has improved considerably over the years.”
  2. The determination of first-class and second-class passenger fares for transatlantic ocean transportation involves problems of (a) joint or related cost, (b) related demand, and (c) discriminatory pricing. Discuss first in what ways these three phenomena are involved here; then formulate a research project to obtain the factual information required for an evaluation of the cost relationships and demand relationships prevailing in the case of two-class passenger ships; and finally state the criteria for judging whether the actual rate differential implies conscious discrimination in favor of first-class passengers, conscious discrimination against first-class passengers, wrong calculation and faulty reasoning on the part of the shipping lines, or any other reason which you may propose.

*  *  *  *  *  *  *  *  *  *  *  *  *  *  *

PART II
June 4, 1956, 2-5 p.m.

Answer three questions, at least one from each group.

Group I.
  1. There is a running debate on the question whether trade unions are labor monopolies. This debate obviously turns on the meaning of monopoly and on what effects union have had on their members’ wages, output, and conditions of work. Give both sides of the argument.
  2. Write an essay on the demand for labor.
  3. Write down everything you know about the incidence of unemployment among various classes of workers and about the fluctuations of unemployment over time. Discuss some of the problems of developing a workable concept of unemployment. Indicate whether the statistical behavior of unemployment throws any light on its causation.
Group II.
  1. What is a “public utility”? According to accepted regulatory principles, how are the “proper” net earnings of a utility company determined? And, finally, what factors are considered in setting an “appropriate” rate structure?
  2. What is the major purpose of the Sherman Anti-Trust Act of 1890? What are some of the more significant problems in determining what constitutes “restraint of trade”? What tests would you apply? Why?
  3. Analyze the economic effects of a corporate income tax. Be as comprehensive as you can.
  4. What are flexible agricultural price supports? Explain how they are determined and applied. Evaluate their use in the light of reasonable alternatives.

*  *  *  *  *  *  *  *  *  *  *  *  *  *  *

PART III
June 5, 1956, 9-12 a.m.

Answer three questions, one from each group.

Group I.
  1. Describe briefly Schumpeter’s theory of economic development, and comment upon the possibility of testing it empirically.
  2. Describe briefly Keynes’ general theory of employment, interest and money; state its assumptions, structure, and conclusions; and evaluate it critically in the light of more recent theoretical and empirical findings.
Group II.
  1. What characteristics of economic cycles would you consider important in a statistical study of business cycles?
  2. In the study of long-term trends, what criteria would you use in constructing index numbers of production?
  3. What measures of economic growth of nations would you us? Consider carefully the various characteristics that you would deem indispensable in measurements of this sort.
Group III.
  1. Give a brief definition, explanation and illustration for each of the following:
    1. variance;
    2. confidence interval;
    3. coefficient of regression;
    4. coefficient of correlation;
    5. coefficient of determination;
    6. regression line.

[Note: Indicate where you have confined yourself to simple, linear correlation.]

  1. Write an essay on statistical inference by means of the following three techniques:
    1. chi square;
    2. analysis of variance;
    3. multiple regression.

Indicate the types of problem in which they are used, and how each type of problem is handled.

*  *  *  *  *  *  *  *  *  *  *  *  *  *  *

PART IV
June 5, 1956, 2-5 p.m.

Answer four questions, one from each group.

Group I.
  1. Political arithmetic is a term that is applied to certain writings that appeared from roughly 1675 to 1800. What gave rise to such writings? What were the contributions of the different members of the “group”? Why should Political Arithmetic be given a terminal date?
  2. Discuss Quesnay’s Tableau Économique, Do you see in it anything of significance for the subsequent development of economic theory?
  3. Present arguments for the contention that J. B. Say was far more than “a mere disciple of Adam Smith”.
Group II.
  1. Discuss the relations between the English economic literature of the first half of the 19th century and the events, conditions, and general ideas of that time.
  2. Select three episodes in American economic history, and use your knowledge of economic theory to explain them.
Group III.
  1. Analyze the economic effects of a large Federal debt. Be as comprehensive as you can.
  2. At one time or another each of the following has been proposed as the proper objective or goal of monetary policy: (1) The stabilization of the quantity of money; (2) The maintenance of a constant level of prices; (3) The maintenance of full employment.
    Explain for each policy objective (a) what it means, that is, exactly what in “operational” terms might be maintained or stabilized; (b) how the objective could be achieved, that is, what techniques could be used to achieve it; and (a) the difficulties with or objections to the proposal.
  3. Irving Fisher and others have proposed that all bank be required to hold 100% reserves against their deposits. This was designed to prevent bank failures and, more important, to eliminate the perverse tendency of money to contract in recessions and expand in booms.
    Explain whether the proposal would have the effects claimed for it, and if so, why, and discuss what other effects it might have.
Group IV.
  1. Discuss the “law of comparative advantage” in international trade.
  2. Discuss “currency convertibility”.
  3. Discuss the “transfer problem”.
  4. Discuss the “optimum tariff”.
  5. Discuss the “foreign-trade multiplier”.
  6. Discuss alternative concepts of the “terms of trade”.
  7. Discuss the “effects of devaluation upon the balance of trade”.

*  *  *  *  *  *  *  *  *  *  *  *  *  *  *

Source: Johns Hopkins University. Eisenhower Library. Ferdinand Hamburger, Jr. Archives. Department of Political Economy Series 5/6.  Box No. 6/1. Folder: “Comprehensive Exams for Ph.D. in Political Economy, 1947-1965”.

Image Source: Fritz Machlup in an economics seminar. Evsey Domar visible sitting third from the speaker on his right hand side. Johns Hopkins University Yearbook, Hullabaloo 1956, p. 15.

Categories
Economics Programs Economist Market Economists Johns Hopkins Kansas Sociology

Kansas. Birth of seminary of historical and political science. Blackmar, 1889

 

A 35 year old Johns Hopkins University Ph.D., Frank Wilson Blackmar, was appointed professor of History and Sociology at the University of Kansas starting in the fall semester of 1889. He joined  the history and civics professor James Hulme Canfield to establish a joint seminary of historical and political science in Lawrence, Kansas. The seminary was to serve as a social scientific laboratory following the model of historical seminaries established earlier in German universities and later transferred to North American universities such as Johns Hopkins during the last third of the 19th century. Blackmar’s Ph.D. subjects were History, Political Economy and Literature and he taught a broad range of courses in political economy, sociology, cultural and intellectual history, as well as social policy at Kansas. He wrote textbooks for both economics and sociology but he eventually left economics for what he must have perceived to be the virgin fields of sociology, a career path similar to that taken by Franklin H. Giddings at Columbia. In 1919 he served as president of the American Sociological Association. 

Frank W. Blackmar served his university for forty years until being unceremoniously shown the door to retirement by his department in 1929. He was even forced to suffer the indignity of witnessing his own venerable sociology textbook dropped for a younger competitor.

Still Blackmar is of interest to us as one of the first generation wave of newly minted Ph.D.’s who were in search of their scientific fortune across the vast expanse of the United States at the end of the 19th century. He also serves as a reminder that the disciplinary wall between economics and sociology was then little more than a speed-bump compared to the well-fortified border today.

This is a long post so I provide the following intrapost links for ease of navigation:

___________________________

Frank Wilson Blackmar

1854. Born November 3 in West Springfield, Pennsylvania.

1874. Graduated at the Northwestern State Normal School at Edinboro, Erie county, Pennsylvania.

1874-75. Taught at West Mill Creek school at Erie.

1875-78. Taught in California public schools.

1878. Enrolled at the University of the Pacific (San Jose, California).

1881. Ph.B. with honors from the University of the Pacific.

1881-82. Taught mathematics in the San Jose High School.

1882-86. Professor of mathematics in the University of the Pacific

1884. A.M. in mathematics and literature from the University of the Pacific.

1885. Married Mary S. Bowman, daughter of Rev. G.B. Bowman, of San Jose.

1886-89. Graduate student and fellow of Johns Hopkins University.

1887-88. Instructor in history.
1888-89. Fellow in history and politics.
1889. June 13. Awarded Ph.D. Thesis: “Spanish Colonization in the Southwest.” Subjects: History, Political Economy and English.  Source: Johns Hopkins University, University Circulars, July 1889, p. 97.

1889-1929. University of Kansas.

1889-1899. Professor of history and sociology.
1889. Course in political economy. Thirteen students enrolled (University Kansan, September 27, 1889, p. 1)
1890. Course “Elements of Sociology” introduced.
1893. Course “Status of Woman” introduced.
1897. Course “Questions of Practical Sociology” introduced.
1897. Course “Remedial and Corrective Agencies” introduced.
1899-1912. Professor of sociology and economics.
1912-1926. Professor of sociology.
1899-1926. Head of the Department of Sociology.
1896-1922. Dean of the Graduate School
1929. Retirement forced at age 74 after 40 years of service to the University. His request to continue full-teaching and full-salary until June 1930 was denied.

1900-02. President of the Kansas Conference of Social Work.

1919. Ninth president of the American Sociological Society

1931. March 30. Died from influenza in Lawrence, Kansas.

Books, monographs, reports

The Study of History and Sociology. Topeka: Kansas Printing Office, 1890.

Spanish Colonization in the Southwest. Baltimore: Johns Hopkins Press, 1890.

The History of Federal and State Aid to Higher Education. U.S. Bureau of Education, Circular of Information No. 9, 1890. Contributions to American Educational History, edited by Herbert B. Adams. Washington, D.C.: Government Printing Office, 1890.

Spanish Institutions in the Southwest. Baltimore: The Johns Hopkins Press, 1891.

The Story of Human Progress, 1896.

Higher Education in Kansas. U.S. Bureau of Education, Circular of Information No. 2, 1890.

Economics. Topeka, Kansas: Crane & Company, 1900.

Spanish Colonial Policy. Publications of the American Economic Association. Vol. 1 (3), 1900, pp. 112-143.

The Study of History, Sociology, and Economics. Topeka, Kansas: Crane & Company, 1901.

The Life of Charles Robinson, the First Governor of Kansas. Topeka, Kansas: Crane & Company, 1902.

The Elements of Sociology. New York: The MacMillan Co., 1905.  [In the Citizen’s Library series ed. by Richard T. Ely]

Kansas: a cyclopedia of state history, embracing events, institutions, industries, counties, cities, towns, prominent persons, etc. (2 vols.) edited by Frank W. Blackmar. Chicago: Standard Pub. Co. Volume I;  Volume II.

Economics for High Schools and Academies. New York: The Macmillan Company, 1907.

Report on the Penitentiary to Governor Geo. H. Hodges. Topeka, Kansas: Kansas State Printing Office, 1914.

Outlines of Sociology, with J.G. Gillin. New York: The MacMillan Company, 1915. [Published in series: Social Science Textbooks, edited by Richard T. Ely. Note: Ely’s own contribution to the series bears the analogous title “Outlines of Economics”]

First edition (1915);
Revised edition (1923);
Third edition (1930).

History of the Kansas State Council of Defense. Topeka, Kansas: Kansas State Printing Plant, December 1920.

Lawrence Social Survey (joint with Ernest W. Burgess). Topeka, Kansas: Kansas State Printing Plant, 1917.

Justifiable Individualism. New York: Thomas Y. Crowell Company, 1922.

History of Human Society. New York: Charles Scribner’s Sons, 1926.

___________________________

Newspaper accounts regarding Blackmar’s appointment

PROF. BLACKMAR SELECTED.
A Good Man Elected to the New Chair of History and Sociology.

At the last meeting of the board of regents of the University, Prof. James H. Canfield recommended that instead of having an assistant as fixed by the legislature, a new chair be created to be known as History and Sociology. Prof. Canfield laid out the work to be pursued by each chair and two chairs were created by the regents as advised by him and he was allowed to have his choice and he took American History and Civics.

To-day the regents, after careful consideration of all the applicants, selected Prof. Frank W. Blackmar, who is taking an advanced course at Johns Hopkins University. Prof. Blackmar is a man of experience and is a graduate of the University of the Pacific at San Jose, Cal. For several years he has been professor of mathematics at that place. The following letter to the regents bears Mr. Blackmar good recommendations:

 

JOHNS HOPKINS UNIVERSITY,
Baltimore, Md.

“The best man I can suggest for your purpose is Mr. F. W. Blackmar, our senior fellow in History and Politics. He was for some years professor in a California college before coming here and has just received an offer of $1500 to go to Mills College in that state. He used to receive $2000, but deliberately threw up a good place in mathematics for the sake of studying history. He is a man of fine character and ability with lots of hard sense and good tact, withal a good speaker and writer. I have employed him upon the most important of all the government monographs, the Relation of Federal and State Aid to Higher Education, a work covering the financial history of education in thirty-eight states. His report has just been accepted in Washington and will do Blackmar great honor. In fact he can get almost anything he wants after that report is published. You will be lucky if you catch him early and you will have to give him all the law allows. I shall recommend Blackmar to the vacancy arising at Bryn Mawr, where Woodrow Wilson used to be, if I am asked to nominate. Blackmar is married, has had experience as a co-educator, and has served as an assistant here, as well as a popular lecturer to workingmen. I have just answered three applications for professors, but have given you the best man

Very truly,
H. B. Adams.

 

With Professors Blackmar and Canfield in the political history department of the University, that department is sure to become one of the most attractive in the University.

Prof. Blackmar is a protectionist, a Republican and a member of the M. E. [Methodist Episcopal] church.

The “Athens of Kansas” welcomes Prof. and Mrs. Blackmar to her midst and we trust that they will find Lawrence a pleasant place in which to live. The success of securing such an able man is due largely to Prof. Canfield and Regent Spangler and the University is to be congratulated upon the new accession to the already strong faculty.

Source: The Evening Tribune, Lawrence, Kansas. Wednesday, May 8, 1889, p. 3.

*  *  *  *  *  *  *  *  *  *  *  *  *

The New Chairs.

The University has taken so many strides towards the front during the past four or five years, that each step has ceased to attract special notice. But a change has just been made which deserves mention, and which has already attracted wide-spread attention and favorable comment.

For several years Professor Canfield has urged a division of his chair, that broader work might be offered in History and in Citizenship. The Board has never been able to meet the necessary expenses of such enlargement, and the work has been carried or driven toward success under many embarrassments.

But now the Regents find the funds on hand for a new chair, and have determined to establish it in this department. Accordingly a special committee has been in consultation with Professor Canfield, and together they have elaborated courses that are peculiarly attractive.

At his own request Professor Canfield retains the work in American History and Civics, which will hereafter be the title of his chair. American History is the favorite option. “Constitutional and Political History of the United States,” elaborated and given daily instead of three times a week. This work absorbs “Colonial History,” “Finance and Diplomacy of the Revolution,” and the “Federalist.” In addition to this will be offered work in Constitutional Law, Public Finance and Banking, Local Law and Administration, and International Law and Diplomacy.

The second chair will be History and Sociology.

It is not possible to say now who the new Professors will be, nor what work will be offered. But the two chairs will work together—the work of one really preparing for that of the other, and together they will make a strong team.

This division of the old chair gives just twice the latitude in choice of options and elections, and the number of students eager to avail themselves of this opportunity is very large.

LATER.

Prof. Frank W. Blackmar, formerly a Professor in the University of the Pacific, and at present a fellow in Johns Hopkins, has been appointed to the chair of History and Sociology, which was recently created by the division of Prof. Canfield’s work Prof. Blackmar comes with the best of recommendations, and will be a strong addition to the faculty.

Source: University Times, Lawrence, Kansas. Friday, May 10, 1889, p. 2.

*  *  *  *  *  *  *  *  *  *  *  *  *

Prof. F. W. Blackmar.

The rumor as to the appointment of Prof. F. W. Blackmar of Johns Hopkins University has been proven true. Through the kindness of Regent Spangler the Journal is enabled to print the following letter concerning Mr. Blackmar:

JOHNS HOPKINS UNIVERSITY,
BALTIMORE, Md., April 13, 1889.

To the Trustees of Kansas State University—Gentlemen:

Allow me strongly to recommend for your new chair of History and Sociology, Mr. F. W. Blackmar, our Senior Fellow in these subjects. He was for four years Professor of Mathematics in his alma mater, the University of the Pacific, where he proved such a good administrative officer that at one time he served as the deputy of the President. We have thought so highly of his ability as a teacher and as a manager of young men that last year we put him in charge of a large class in History in our undergraduate department. He taught the class to our entire satisfaction. This year as Fellow he has not been allowed to teach, but has given his entire attention to original investigation. Besides writing a scholarly thesis on “Spanish Colonization in the Southwest,” based upon Spanish and other original authorities, he has completed under my direction a most elaborate government report on “The Relation of Federal and State Aid to the Higher Education,” or a financial history of American Colleges and universities in so far as they have been supported or assisted by government appropriations. This report, I am confident, will give Mr. Blackmar a national reputation, for it will meet the needs of every State board of trustees and of all superintendents of education in these United States. In addition to this government and university work, Mr. Blackmar has lent a hand in various popular lecture courses which I have instituted here in Baltimore. I append the printed outlines of one or two of his lectures. He is a man who can go before the people, if necessary, and make himself understood on practical questions. He takes a strong interest in social science, or questions affecting the public health and welfare, such as Sanitation, Charities, the Relation of the State and City to the care of Paupers, the Insane, the Blind, the Deaf and Dumb, etc. If you should see fit to appoint Mr. Blackmar to Historical. Economic and Social Science, it would be wise to encourage him during the coming summer to visit the leading charitable institutions of New York and Massachusetts, and to acquire a practical knowledge of the best methods, from interviews with men like Mr. Brockway, of the Elmira Reformatory, and with Mr. F. S. Sanborn, long secretary of the State Board of Charities in Massachusetts, and at one time lecturer upon these subjects at Cornell University. There is a great field here for a well-trained University man. With knowledge of the best experience of the world he can promote the usefulness and economy of charitable institutions throughout an entire state or city. The Johns Hopkins University is pushing men into this new field. Two of our graduates in succession have served as Secretary of the Organized Charities of Baltimore. Another has similar position in Brooklyn. A fourth has just been made Secretary of the New York State Charities Aid Association, an office which brings him into active relation with all the charitable institutions of both city and state. I emphasize these facts because they show the practical bearings of Social Science when properly represented in a University.

Let me say, in conclusion, that Mr. Blackmar is a young man of excellent moral character, a Christian gentleman, married and in good health, although just now a little overworked while preparing for his degree as Doctor of Philosophy. He is perfectly safe in all economic and social questions and is naturally endowed with a good stock of common sense.

Very respectfully recommended,
H. B. Adams.
In charge of the Department of History and Politics.

Mr. Blackwar is 34 years of age and a native of Erie county, Pa. In 1874 he graduated at the Northwestern State Normal school at Edinboro, Erie county, Pa.; the following year he taught the West Mill Creek school at Erie, at the same time carrying on studies preparatory for college. In the autumn of 1875 he went to California and there engaged in teaching in the public schools for a term of three years; in 1878 entered the University of the Pacific, San Jose, California, and graduated from that institution in 1881 receiving the degree of Ph.B. The following year he engaged in teaching mathematics in the San Jose High School. In 1882 he was called to the chair of mathematics in the University of the Pacific which he filled acceptably for a term of four years. In 1884 he received the degree of A.M. on account of work done in mathematics and literature.

In the following year he was married to Miss Mary S. Bowman, daughter of Rev. G. B. Bowman, of San Jose.

In 1886 he entered Johns Hopkins University, when he was appointed instructor in 1887 and fellow in history and politics in 1888.

Source: Lawrence Daily Journal, Lawrence, Kansas. Friday, May 10, 1889, p. 3.

*  *  *  *  *  *  *  *  *  *  *  *  *

Prof. Blackmar Elected.

Tuesday the board of regents after the consideration of all the applicants elected Prof. Frank W. Blackmar, who is now taking an advanced course at John Hopkins, to fill the associate chair in the history department. Prof. Blackmar is a graduate of the University of the Pacific, a republican, a prohibitionist, a Phi [Kappa] Psi. The following letter to the regents bears Prof. Blackmar good recommendations, and the Courier bids him welcome.

JOHNS HOPKINS UNIVERSITY,
Baltimore, Md.

“The best man I can suggest for your purpose is Mr. F. W. Blackmar, our senior fellow in History and Politics. He was for some years professor in a California college before coming here and has just received an offer of $1500 to go to Mills College in that state. He used to receive $2000, but deliberately threw up a good place in mathematics for the sake of studying history. He is a man of fine character and ability with lots of hard sense and good tact, withal a good speaker and writer. I have employed him upon the most important of all the government monographs, the Relation of Federal and State Aid to Higher Education, a work covering the financial history of education in thirty-eight states. His report has just been accepted in Washington and will do Blackmar great honor. In fact he can get almost anything he wants after that report is published. You will be lucky if you catch him early and you will have to give him all the law allows. I shall recommend Blackmar to the vacancy arising at Bryn Mawr, where Woodrow Wilson used to be, if I am asked to nominate. Blackmar is married, has had experience as a co-educator, and has served as an assistant here, as well as a popular lecturer to workingmen. I have just answered three applications for professors, but have given you the best man

Very truly,
H.B. Adams.

Source: The University Courier, Lawrence, Kansas. Friday, May 10, 1889, p. 2.

*  *  *  *  *  *  *  *  *  *  *  *  *

FRANK W. BLACKMAR

An extended sketch of Prof. Blackmar was given in The Courier last Spring, but for the benefit of the new students we reproduce a part of it. Prof. Blackmar is a native of Pa., and graduated from the Northwestern Normal School in 1874. He then went to California and taught a few years in the Public Schools of that State. He then entered the University of the Pacific, and graduated with honors with the class of ’81. He taught in the San Jose High School, and was then called back to the University of the Pacific to fill the chair of mathematics. This position he held until 1886, when he resigned to pursue a post graduate course in Johns Hopkins. During the year 1887-8, he was an instructor in History at that institution, and at the time of his election to the chair of History and Sociology in the University last spring, was a Fellow in History and Politics at Johns Hopkins. He is a member of Phi [Kappa] Psi Fraternity, having joined that organization while a student at the University of the Pacific. He took his degree of Ph.D. last June, at Johns Hopkins, the subjects covered in his course being History, Political Economy and English.

Source: The University Courier, Lawrence, Kansas. Friday, August 16, 1889, p. 2.

___________________________

Department of History, Politics and Sociology—A Circular Issued Covering the Work in that Department.

The department of history in the State University has just issued a circular covering the work in that department. By the division of the chair of history and the election of an additional professor in that department the long wished for equalizing of the course was attained and under the name of American History and Civics and History and Sociology the University presents as strong and comprehensive course in that line of college work as any other college in the country. In order that our readers may know for themselves the extent of this course and also the division of the work between Prof. Canfield and Prof. Blackmer we print the circular entire.

HISTORY, POLITICS AND SOCIOLOGY.

The following statement covers the work of the last two years of the University course, and is made in answer to many inquiries received by the instructors in charge of these topics.*

Instruction in History, Polities, and Sociology is given by means of lectures, recitations, discussions, conference, and personal direction in study and research. Special pains are taken to facilitate the use of the University library by students taking these topics; authorities closely connected with the work in hand being withheld from general circulation, and rendered more available by carefully prepared card indexes.

AMERICAN HISTORY AND CIVICS. — JAMES H. CANFIELD.

American History. — Instruction is given daily for two years in American History. The course has been prepared with especial care, with the thought that a thorough knowledge of the origin and development of the Nation is one of the most essential conditions of good citizenship. Marked attention is given to social life and institutional and industrial development; to the financial experiments of the general government, and to diplomatic relations; to the failure of the confederation, the struggle for the constitution, and to the text of the constitution itself; and to the constitutional and political history of the Union from 1789 to the present. For this the library now offers special facilities, in a complete Congressional Record, from the first Continental Congress to the present (including the Secret Journals and Diplomatic Correspondence), a complete set of Niles’ Register, and in a large collection of other public documents.

Local Administration and Law. — Lectures three times each week during the first term,† covering the management of public affairs in districts, townships, counties, cities, and States. This course is intended to increase the sense of the importance of home government, as well as to give instruction in its practical details.

Public Finance and Banking. — Lectures twice each week during the first term, on National, State, and municipal financiering; and on theoretical and practical banking, with the details of bank management.

Constitutional Law. — Lectures three times each week during the second term, on the constitution of the United States; with brief sketches of the institution and events that preceded its adoption, and with special attention to the sources and methods of its interpretation.

International Law and Diplomacy. — Lectures twice each week during the second term on the rise and growth of international law, and on the history of American diplomacy.

In all this work constant effort is made to determine the historic facts (as opposed to mere theorizing), to secure a fair presentation of opposing views, to promote free discussion and inquiry, and to encourage as complete personal investigation of all authorities as the University library permits. This method is thought to furnish the best conditions for sound opinion and individual judgment, while controlling neither.

HISTORY AND SOCIOLOGY.
— FRANK W. BLACKMAR.

The aim in the following courses is to give a comprehensive knowledge of the great topics of history, and to investigate general social, political, and economic phenomena and theories — especially those of Europe.

Instruction will be given daily throughout the first term, as follows:

English History. — This course embraces a careful study of the English people and the growth of the English nation, including a general survey of race elements, and of social and political institutions.

The Intellectual Development of Europe. — A course of lectures tracing the history and philosophy of intellectual progress from early Greek society to modern times.
Particular attention is given to the influence of Greek philosophy, the Christian church, the relation of learning to liberal government, and of the rise of modern nationality.

Political Economy. — The fundamental and elementary principles will be discussed, and will be elaborated by descriptive and historical methods. A brief historical sketch of Political Economy may be given at the close of the course.

The second term’s work includes the following courses:

Institutional History. — Lectures three times each week, on Comparative Politics. The history of Germanic institutions will constitute the main body of the course.

The Rise of Democracy. — Lectures twice each week, on the rise of popular power and the growth of political liberty throughout Europe.

Elements of Sociology. — Lectures three times each week, on the evolution of social institutions from the primitive unit, the family; including a discussion of the laws and conditions which tend to organize society. The latter part of the course will be devoted to the elements of modern social science as preliminary to the consideration of the problems of the day.

Land and Land Tenures. — Twice each week. The course will begin with a discussion of the Roman land question and extend to the Feudal land systems of France and England, and thence to the consideration of modern land tenures of Great Britain and of the United States.

Practice Course in Economics. — A full term’s work applied in economics and in the elements of social science; consisting of conferences, discussions, practical ob-servation, and the preparation of a thesis of not less than twenty thousand words on some special topic selected by each student

All general correspondence should be addressed to the Chancellor of the University; special correspondence, to either of the instructors named in this circular.

*During the first two years of the University course, students have the subjects usually required in college courses — though with choice between four lines of work. (See University Catalogue.)

†The University year is divided into two terms, of equal length.

Source: Lawrence Daily Journal (Lawrence, Kansas), Sunday, July 14, 1889, p. 3.

___________________________

Seminary of Historical and Political Science.

Announcing the new Seminary

The Political Science Club has been succeeded by the Seminary of Historical and Political Science. This new society has been organized by Profs. J. H. Canfield and F. W. Blackmar. The membership of the society is limited to the department of History and Political Science, students having two or more studies in that department being active members and those having less than two studies being associate members.

This new association will embrace all of the best features of the Political Science Club, besides several new features. From his two years’ experience with the Political Science Club, Prof. Canfield is able to accept only those features that have proven to be practical. Under the new management the Seminary is expected to be even more interesting and valuable an adjunct to the department, in the future, than the Political Science Club has been in the past.

Source: University Kansan (Lawrence, Kansas), Friday, September 27, 1889, p. 2.

*  *  *  *  *  *  *  *  *  *  *  *  * 

First Annual Symposium of the Seminary of Historic and Political Science of K. S. U.

A short time ago invitations were sent out to the members of the Political Seminary, and last evening a goodly number of both active and corresponding members assembled for the first annual reception and banquet. The guests were received by Prof. J. H. Canfield, director, and Prof. Blackmar, vice-director of the Seminary; and at 8:45 the line of march was taken up for room 15, which served for a banquet hall as well as a lecture room.

Here an excellent repast was served by young lady students of the University, after which Director Canfield announced the Symposium proper. The director spoke at some length of the work accomplished in the past year, giving a list of the more important papers presented, and announced that for next year papers on subjects of interest were promised by Geo. R. Peck, of Topeka; Frank H. Betton, labor commissioner of Kansas; Judge Humphrey, of Junction City; Judge Emery, D. S. Alford, Rev. Ayers, Charlie Scott and Dr. Howland.

Prof. Canfield then introduced Vice-Director Blackmar, and asked him to compare the work done by the seminary here with the same class of work in eastern colleges. Prof. Blackmar gave a short account of the present mode of studying history and political economy, saying that it was of recent date. Comparisons with Yale, Johns Hopkins and the University of Michigan, show that the work here is as thorough as at any of those institutions. The study of the Science of History has risen into prominence, as has the study of the natural sciences, and furnishes as good mental training as do the languages or even mathematics.

At the close of Prof. Blackmar’s speech Prof. Canfield announced the real topic of the occasion, “The University in its Relation to the People,” and called on Gov. Robinson to tell of the early struggle for a university in Kansas. The governor then told in his own happy manner of the early endeavors to secure a university in Kansas, of the first faculty and how it was selected for policy’s sake, of the work that the regents had to do even in the first years of the University. His hope that the present director of the seminary would never leave the University was heartily applauded.

The time having arrived when three minute speeches were in order, Prof.

Canfield called on Mr. H. F. M. Bear to talk on the “Influence of the University in the Community.” Mr. Bear opened with a story, and when that was finished so were his three minutes.

“What a University Course in Worth to the Bar of the State” was responded to by Judge Humphrey; the judge said “That a thorough collegiate education is becoming more and more recognized as a necessity in the lawyer’s profession; that the most important function of a state school is to equip men for good, honest lawyers.”

“The University Man in Politics” was discussed by A. L. Burney, of the class of ’90, “The true duty of the University graduate in politics is to be a leader, following the teachings of the golden rule.”

Colonel O. E. Learnard, in responding to “the University man” in connection with the press, said: “It was not a good plan to mix a University training with newspaper work, but that men should graduate from the University into the newspaper profession.”

Prof. Canfield, introducing the next speaker, congratulated the University in having begun right in the matter of co-education.

Miss Hunnicut, a post-graduate student in Political Science, spoke on “Post-Graduate Work, the link between the University and practical life;” thought the course too short, should be two years instead of one — this was the only opportunity offered the student to do original work.

“University boys’ outing life” was assigned to C. E. Esterley. Mr. Esterley declared that the University boys were always successful after leaving school.

“What the University can and does do for women,” was discussed by Miss Reasoner — class of ’90. Miss Reasoner is a pleasing speaker and was listened to with close attention,

Prof. Blake in speaking on “the University and applied science,” said that everything in K.S.U. depended on the crops in Kansas, and as the crop prospect was good this year, so was the outlook for applied sciences hopeful in our University. The object in giving our young men instruction in the shops was not that they might be laborers, but directors of our great industrial enterprises in the West.”

This closed the program of a most successful meeting, and Prot. Canfield then declared the assembly adjourned for one year.

Those present were: Prof. J. H. Canfield, director; Prof. F. W. Blackmar, vice-director; Misses Lockwood, Dunn, Spencer, Reasoner and Hunnicutt; Judge Humphrey, Gov. Robinson, Dr. Howland, Rev. Mr. Ayres, B. W. Woodward, D. S. Alford, Col. O. E. Learnard, Prof. Blake, and Messrs. Chapman, Esterly, Liddeke, Slosson, Burney, Mushrush, Bear, Roberts, Morse, Hill, Wilmoth.

Source: Lawrence Daily Journal (Lawrence, Kansas), Thursday, June 5, 1890, p. 4.

___________________________

Blackmar on place of political economy 

ECONOMIC POLITICS. — One branch of political economy falls directly within the scope of history, and this is what may be termed economic politics, or that part of political economy which has to do with the action of the state concerning economical development. This has been called “Historico-Political Economy,” as treated by the historian. It deals less with economic life as a philosophy, and more with the practical affairs of economic legislation. As such it might assume the German name of “National Economy,” only that it would include more than is here intended. It is a separate study from the science of Political Economy as now constituted. However, in the earlier conditions of the science, and to a certain extent now among some French and German writers, political policies are confused with the science of political economy.

Within the scope of economic politics should be grouped those social and economic movements which have been directly connected with the political changes that have taken place in states. Some of the so-called political institutions have their direct cause of existence, in social or economic movements. The so-called new school, or, what is more explanatory, the “historical school” of political economists, in contradistinction to the old or “deductive” school, base their operations upon historical conditions rather than upon a priori arguments. Consequently, the association of political economy with the study of history has become common. It is true, on the one hand, that science of political economy that struggles with a priori principles, ideal men, ideal nations, and ideal conditions, was released from many of its defects when a careful search into historical conditions was made. On the other hand, there is a politico-economic history of nations which may be incorporated with the study of history proper, and still allow Political Economy to retain its own province undisturbed. It is this phase of political history which should come under the head of economic politics. The study of Political Economy as an independent science will be treated of under that heading.

*  *  *  *  *  *  *  *  *  *  *  *  *

Such was the condition of the study of history in the American college up to a recent period, that the dull, dry conning of the facts of universal history with the chief idea of knowing the facts of the world’s history only to forget them, was the recognized process. President Adams tells us that during the first two centuries of the existence of Harvard College, the study of history consisted in spending one hour at eight o’clock on Saturday mornings in the hearing of compositions and the reciting of history, both ancient and modern. In 1839 a special chair for the study of history was endowed for the college, yet it was not until 1870 that there was any real change in the method pursued of conning history. At that time two men were employed, where before one man did all of the work. From this time there was rapid improvement. The condition in Yale and in Columbia was not much better than that in Harvard; in Yale the entire services of one man were not required until after 1868, to teach history, and it was not until 1877 that another man was put into the field.

In 1857 President White, of Cornell, instituted the study of history in the University of Michigan, and used the historic method employed in Germany with some modifications. This method was adopted in Cornell in 1870, and in Johns Hopkins in 1876, at the commencement of its career. With these beginnings a rapid progress has been made towards the treatment of history from a scientific standpoint. From this time the best institutions of America abandoned the old, dull process of memorizing and forgetting the facts of history without making good use of those facts. But this progress is not equal to the progress made in the old-world institutions in the organization and arrangement of courses and the number of separate fields of study. The methods used are somewhat the same.

Modern methods of historical teaching have for their chief points the systematic work of the student under the intelligent direction of the instructor. The process involves an investigation of materials, a search after the truth, a study of particular phases of historical truth, a comparison and classification of material, and an analysis of results. History is to be studied because it is interesting, and to be followed for the truth it will yield. In all of this the facts of history must not be ignored, nor the careful reading of standard authorities neglected. But the instruction works upon the principle that a person engaged in an interesting pursuit of the truth of history will retain by real knowledge of the subject the facts which if learned by rote without understanding would soon leave him.

*  *  *  *  *  *  *  *  *  *  *  *  *

The MODERN SEMINARY furnishes a means of bringing together those most interested and most advanced, for the special study of subjects in history or in political and in social economy. This method, now almost universal in the foremost institutions, is of German origin, and constitutes the germ of the modern method. The seminary had its origin with the class taught by Leopold von Ranke, and from that time has been greatly improved in Germany, and extensively adopted in America. The seminary represents the historical laboratory, and each meeting should be a clearing-house of the actual work done. The object of the seminary is to develop individual thought and investigation, and to test the same by criticism and discussion. Another beneficial result will be the development in a practical way of the best methods of study. We have laboratory work in physics, chemistry, and in most of the natural sciences; if history is to be taught as a science, it must not ignore this great means of investigation. Its work may not always be original, for the word original should be used with much care in its application to any study. It must be sufficiently individual and independent that the student may verify truth by his own investigation, and learn to exercise his own judgment concerning the materials before him. The undergraduate courses in chemistry or physics seldom go beyond this in their laboratory work. The seminary is an association of individuals coöperating in the pursuit of historical truth, using scientific methods in study, research, and presentation. It should represent the highest and best work of any department or group of departments working on kindred subjects.

But whatever methods are pursued, it must be kept in mind that there are scientific processes involved, and scientific results must be expected. The chief benefits to be derived from the study of history, or of the different branches of history and sociology, are similar to those of all other sciences.

*  *  *  *  *  *  *  *  *  *  *  *  *

Professor R.T. Ely wrote an Introduction to Political Economy which was more or less sociological in its nature, and which assumed that Political Economy was a branch of Sociology. Subsequently a controversy arose as to the relative position of Economics and Sociology, which has been finally settled by Sociology taking and maintaining an independent position in the category of social sciences. While nearly everything relating to society has been called, at different times, Sociology, there is to-day a well-established body of knowledge, well-defined principles, and a distinct boundary of the science of Sociology.

A word must be said about the treatment of what is known as “social science” in a peculiar way, as if the only province of sociology was to care for broken-down and imperfect society; and that sociology has to deal only with social problems, and not with the rational development of human society. It must be acknowledged that the value of the study of charities and corrections cannot be overestimated, and that as representative of the position of a certain phase of social disorganization, the study of these is invaluable. These studies represent the outcrop pings of society, and just as a ledge in the mountains will show by its nature the condition of the original bed, so these parts of disorganized society will show the nature of the true structure. So, also, as it treats chiefly in its scientific methods of the reorganization of society, there is an opportunity offered for the application of the best results of the study of sociology.

*  *  *  *  *  *  *  *  *  *  *  *  *

THE STUDY OF ECONOMICS.

INSTRUCTION in economics has been for many years a part of the regular course of nearly every college and university in the land, and has recently been making rapid advancement in the secondary schools. There has been rather more controversy respecting its scope than of the methods employed in study and instruction. Some have contended that as Economics is an abstract science its scope is narrow, comprising only the body of principles and laws that have been drawn from concrete experience. Others have broadened the subject to include much that rightfully belongs to sociology and political science. Others have adopted the historical method to such an extent as to exclude all scientific nature of the subject, reducing it to a mere relation of facts concerning the industrial affairs of the nation. As usual, extremists may be of service in quickening thought, but they seldom hit upon the correct solution of problems that concern a large number of people. While it is proper and unavoidable to hold to the abstract or deductive political economy, it is also necessary to carry on concrete investigations by the inductive method. Nor must industrial history be neglected, for this makes a strong background for the science and enables the student to approach the subject from a new point of view. If a student will observe the following analysis and obtain a thorough knowledge of the subjects enumerated therein, he will have a fair knowledge of the science of Economics from every essential point of view. This analysis represents the essentials of economics; more would be superfluous and less would be insufficient. True, there are many subjects more or less directly related to economics, such as economic statistics, economic ethics, and economic jurisprudence, but they do not make up the body of the subject as a science.

CLASSIFICATION OF ECONOMICS.

  1. Classification according to the nature and logic of the science:
    1. Pure or abstract Political Economy.
      1. Laws, principles, and theories.
    2. Applied economics.
      1. Verification of laws and principles in concrete economic life.
      2. Practical investigation into economic phenomena, general or special, and classification and deduction of the same.
      3. Consideration of ideal standards and the means of approximating them.
    3. History of economic thought.
    4. Industrial history.
    5. Methodology of the science.
  2. Classification according to agencies:
    1. Private or non-political economics.
    2. Public or political economics.
      1. Public control of industries.
      2. Taxation and finance so far as related to economics.

While this outline carefully followed would give a student a fair knowledge of economics, it is not possible for him to have such knowledge with a narrower scope. Every point of the science is carefully fortified with concrete examples of economic life, and the progress of the industries acquaints one with the causes of changes or the process of economic evolution.

METHODS OF STUDY.

The chief difficulty met by the instructor in economics is to separate the principles and laws of economics from theoretical discussion. Theory of economics may have an important place in the class-room, but it is simply discouraging to find in an ordinary economic library that theory occupies so great a place in nearly all books on the subject. If economics is a science, what are its principles, what are its laws, and what the great body of classified knowledge that makes up the real elements of the science? In beginning the subject, then, it is necessary for the instructor to define carefully the boundary of the science. The student wants to know somewhat definitely the scope and purpose of the subject. If there is a science of economics, he wants to know definitely what is comprised in the body of classified knowledge it represents, and what are the laws and principles involved in its scientific processes.

After determining the scope of the science his next difficulty is in the classification of its subject-matter. This in itself is a difficult question; nor is the difficulty confined to economics, for it abounds in all social sciences and extends to considerable extent in the physical sciences. He will find the logical and comprehensive classification of either economic principles or economic phenomena a most difficult process. If the instructor or student can find the above conditions met in a well-arranged text-book the trouble is half over, for the principles of economic science are not difficult. Such a text-book should contain all of the essentials of the science, and should eliminate all controversial points and theories not yet well founded. For the discussion of theories, the elaboration of special topics, and the consideration of the views of economists, the student, like the instructor, must go to the library. For beginning classes this library should consist of a few carefully chosen books, each with a specific purpose. The library method in economics is largely the composition method, or possibly the compilation method. The student gets a re-statement of the principle of the text or lecture, either from a different point of view or in a more extended discourse. Great care should be taken to prevent a rambling course of reading, which is frequently carried on to the confusion of the student.

After the elements have been fairly well mastered the future work of the student should be on one or more of the great topics in economics, such as Money and Monetary Theories; Banking; Taxation and Finance; Industrial History; History and Theory of Economics: or the student may work on special themes, following them to the utmost limit, such as Capital, Wages, Interest, Labor Organization, Prices, etc.

In all this study the instructor and student must not forget to go to the concrete for verification, for illustration, and, so far as possible, for investigation. He must not forget that economic life and economic society are all about him, and the processes of economic practice, change and growth are to be observed at any time he will take the pains to inquire into their operations.

So long as the operations on the farm, the management of the household, the conduct of the factory, the operation of a bank, and the management of a railroad are ever present, the student from the beginning to the end of his course may find by actual study of the concrete the operation of the laws and processes of economics. Some difficulty will be met in teaching beginners to discriminate between the production of wealth in our economic sense and the technology of wealth-getting. In all concrete investigation this is to be carefully considered. For it is the general processes of production and their effects upon the market and upon society as a whole that interest the economist. Economics will not teach a boy how to carry on agriculture, or manufacturing; it will not teach him how to grow wealthy, except that as he studies finance, taxation, money, banking, production and distribution of wealth, he will have developed a tendency of thought, and an intelligence which would make him a better business man, a better financier, if he puts his knowledge to the proper use. The subjects treated in a general way will prepare a man theoretically if not technically for a business life. And without doubt, universities will eventually develop schools of commerce, trade, banking, business, and public service, which will give a professional and technical education in the great lines of industrial life.

The student must keep his eyes turned constantly upon the economic life around him if he would keep his knowledge from becoming visionary and non-vital. By a careful study of the actual operations of society in regard to questions of wealth and well-being, he will develop a practical knowledge of affairs that will be of service to himself personally and to the public at large. He will also find it convenient and profitable to consider the defects of economic life as compared with an ideal standard of justice, and set up a program of action. It is true that here he enters the field of economic ethics. If he then searches for a remedy for existing evils he enters economic politics. Yet economics as a science cannot be said to have worked out its purpose until it has become utilitarian in its attempt to better social conditions. It will not have done its duty until it inquires what ought to be. It should determine how the economic system of the world might bring a larger measure of justice to men, and plan such measures to be acted upon by the public to bring about a better condition of affairs. Every science must in the ultimate be of practical service to humanity if it has a reason to exist, and economics is especially adapted to render great service to humanity if properly studied and wisely taught.

*  *  *  *  *  *  *  *  *  *  *  *  *

SELECTED REFERENCES.

[History]

ADAMS, C. K. — On Methods of Teaching History.

ADAMS, C. K. — Recent Historical Work in Colleges and Universities of America.

ADAMS, H. B. — Special Methods of Historical Study.

ADAMS, H. B. — New Methods of Study in History.

ALLEN, W. F. — Grades and Topics in Historical Study.

BLACKMAR, F. W. — The Story of Human Progress.

BERNHEIM, ERNST. — Lehrbuch der Historischen Methode.

BURGESS, J. W. — The Methods of Historical Study in Columbia College.

CALDWELL, H. W. — American History Studies.

DIESTERWEG, G. — Instruction in History.

DROYSEN, JOH., GUS. — Grundriss der Historik.

DROYSEN, JOH., GUS. — Principles of History. (Tr. by ANDREWS.)

EMERTON, E. — The Historical Seminary in American Teaching.

FLINT, ROBERT. — The Philosophy of History.

FLING, CHARLES MORROW. — Studies in European History.

FREEMAN, E. A. — Methods of Historical Study.

GETSCHELL, MERLE S. — The Study of Mediæval History by the Library Method.

HALL, G. STANLEY. — Methods of Teaching and Studying History.

HART, ALBERT BUSHNELL. American History told by Contemporaries.

LORENZ, OTTOKER. — Geschichtswissenschaft.

MACE, WILLIAM H. — Method in History.

MAURENBRECHER, WILHELM. — Geschichte und Politik.

[Sociology]

BLUNTSCHLI, J. K. — The Modern State.

CROOKER, J. H. — Problems in American Society.

DE GREEF, GUILLAUME. — Introduction a la Sociologie.

FAIRBANKS, ARTHUR. — Introduction to the Study of Society.

GIDDINGS, F. H. — Principles of Sociology; Sociology and Political Economy.

COMTE, AUGUST. — The Positive Philosophy.

KELLY, EDMOND. — Government, or Human Evolution.

LOTZE, HERMANN. — Microcosmus.

SEELYE, JULIUS H. — Citizenship.

SMALL, ALBION W. — Introduction to the Study of Society.

SMALL, ALBION W. — Methodology in Sociology.

SMITH, R. M. — Statistics and Sociology.

SPENCER, HERBERT. — Principles of Sociology.

SPENCER, HERBERT. — The Study of Sociology.

WARNER, AMOS G. — American Charities.

WARD, LESTER F. — Dynamic Sociology.

WARD, LESTER F. — Outlines of Sociology.

WILSON, WOODROW. — The State.

WRIGHT, CARROLL D. — Statistics in Colleges.

WRIGHT, CARROLL D. — Practical Sociology.

[Economics]

BLACKMAR, F. W. — Economics.

COSSA, LUIGI. — Introduction to the Study of Political Economy.

ELY, R. T. — Outlines of Economics.

ELY, R. T. — The Past and Present of Political Economy.

GIDDINGS, F. H. — The Sociological Character of Political Economy.

INGRAM, J. K. — The History of Political Economy.

SMITH, R. M. — Statistics and Economics.

Source: Frank W. Blackmar, The Study of History, Sociology, and Economics, pp. 7-8, 30-31, 56-58, 66-67, 83-89. Published in the series Twentieth Century Classics, No. 17 (January 1901). Topeka, Kansas: Crane & Company.

___________________________

New Staff, New Names
Rebranding

The New Professors.

The resignation of Prof. James H. Canfield, regretted by all, has led to the reorganization of the work in history and political and social science. The two departments formerly known as those of American History and Civics, and History and Sociology respectively, have been combined into the one department of History and Sociology. This department is in charge of Prof. Frank Wilson Blackmar, Ph.D. To assist in the instruction in this department, the Board has elected F. H. Hodder, Ph.D., to be Associate Professor, and E.D. Adams Ph.D., to be Assistant Professor. Dr. Hodder is taken from the faculty of Cornell University. He has for the last year been pursuing historical studies in the University of Freiburg, Germany. He comes to the University of Kansas with a fine reputation for scholarship and teaching ability. Dr. Adams is a young man, a graduate of the University of Michigan and a brother of Prof. Henry C. Adam’s. Michigan University’s professor of Political Economy and Finance. Dr. Adams comes to the University with many good words from the strong men of eastern institutions.

Source: The Lawrence Gazette (Lawrence, Kansas). Thursday, August 6, 1891, p. 2.

*  *  *  *  *  *  *  *  *  *  *  *  *

New name: Department of History and Sociology (1891)

Since the publication of the last number of Seminary Notes, several important changes have taken place. First, Mr. E.D. Adams was elected Assistant in History and Sociology. Soon after this Professor Canfield resigned his professorship to go to Nebraska. Immediately after accepting his resignation, the Regents consolidated the two historical departments, under the title of History and Sociology, and elected Mr. F. H. Hodder Associate Professor. It necessarily follows that the editorial staff of Seminary Notes has two new men in the place of Professor Canfield. The present editors will carry out the original plan of the publication with such improvements as may be made from time to time.

We are glad to learn of the prosperity of the former director of the Seminary, Chancellor James A. Canfield. The number of students enrolled in the University of Nebraska is thirty per cent, greater than last year. A new Law course has been established in the university. Upon the whole the new Chancellor of Nebraska is doing just what his friends predicted — making a great success of his new work. The University of Nebraska is to be congratulated that it was able to secure such an efficient man as Chancellor Canfield.

[…]

The senior professor [Frank W. Blackmar] in the department of History and Sociology is highly gratified that the Regents of the University have again displayed their wisdom in electing two able men to positions in the department. They are young men of scholarly habits and marked ability. Professor Hodder, Associate in American History and Civics, was born at Aurora, Ill., November 6, 1860. He graduated at Michigan University in 1883, having studied history under Prof. C.K. Adams, and political economy under Prof. H.C. Adams. He was principal of the High School at Aurora. Afterwards he went to Cornell University, where he was instructor and later Assistant Professor in Political Economy from 1885 to 1890. During the last year he has been studying at the universities of Göttingen and Freiburg, under Von Hoist, Conrad and others. He is an able instructor.

Mr. E.D. Adams, Assistant in History and Sociology, was born at Decorah, Iowa, in 1865. He was a student in Iowa College, 1883 to 1885; student in the University of Michigan 1885 to 1887, taking the degree of A.B. in 1887, was principal of the High School at McGregor, Iowa, 1887 to 1888, and student of the University of Michigan for the degree of Doctor of Philosophy, 1888 to 1890. In 1890 he took the degree of Ph.D. Since 1890 he has been connected with the census work on street railways, and since December has held the position of special agent in charge of street railways. He is doing good work in Kansas University.

Source: Seminary Notes published by the Seminary of Historical and Political Science, Vol. I, No. 2 (October 1891), pp. 39-40.

Image Source: Kansas yearbook,The Jayhawker 1901, p. 18. Colorized by Economics in the Rear-view Mirror.
Cf. portrait of Herbert Baxter Adams posted earlier. His master’s look?

Categories
Exam Questions International Economics Johns Hopkins

Johns Hopkins. International Economics Exams. Balassa, 1968-69.

 

This post is able to match the examination questions to the corresponding reading list for one semester of Bela Balassa’s international trade theory course that he taught at Johns Hopkins in 1968-69. Alas, the archival box did not have the reading list for the second semester, but at least the exam questions for the second semester, also transcribed below, give us a good idea of the main course content during the spring of 1969.

I am also delighted to have found a picture of Bela Balassa to replace the one I had found on a webpage that, as it turns out, happens to be of an entirely different Balassa (see note at the bottom of the post for details). Professor M. Ali Khan of Johns Hopkins tipped me off about the previous picture (used in other posts) not being quite right. 

____________________________

Note: the reading list for the fall semester course was transcribed and posted earlier.

EXAMINATION
INTERNATIONAL ECONOMICS 18.641
Thursday, January 16, 1969

Dr. Balassa

  1. Answer two questions (80 minutes)
    1. Discuss the meaning of the expressions “labor” and “capital” in the Heckscher-Ohlin framework and indicate the implications that the recent interpretations of these concepts have for the theory of international trade.
    2. Analyze the relationship between country size and the commodity composition of exports and imports.
    3. Discuss the applicability of alternative theories of specialization to trade among industrial countries.
  2. Answer two questions (80 minutes)
    1. Examine the usefulness of a general equilibrium approach to trade theory.
    2. Consider the implications of introducing intermediate goods in trade models.
    3. Show the applicability of the theory of duopoly and bilateral monopoly to the theory of tariffs.
  3. Answer one question (40 minutes)
    1. State briefly the Stolper-Samuelson and the Rybczynski theorems and indicate the relationship between the two.
    2. What welfare consequences can be derived from the following results if subscript 2 refers to the after-trade and subscript 1 to the before-trade situation:

ΣP2Q2 < ΣP2Q1

ΣP1Q2 > ΣP1Q1

____________________________

Note: the reading list for the spring semester course was not included in the collection of course outlines for the department of political economy in the Johns Hopkins University archives.

Final Examination
International Trade Theory 18.642
May 21, 1969

Professor Balassa

Give approximately equal time to all questions.

  1. Answer two questions.
    1. It has been customary to consider separately internal and external balance and to examine the effects of the use of various policy instruments on each. How can this be reconciled with Johnson’s proposition that “the balance-of-payments is the difference between aggregate receipts and payments in the domestic economy:”
    2. Reformulate the exchange stability problem if the devaluation is regarded as a transfer.
    3. Indicate the effects of a devaluation on the non-merchandise items of the balance of payments.
  2. Answer two questions.
    1. Examine the welfare implications of alternative means for attaining balance-of-payments equilibrium, including devaluation, restrictions on trade, restrictions on capital movement, and domestic deflation.
    2. Milton Friedman has recently argued that the introduction of the two-tier gold market has placed the world on a dollar standard and thus the United States no longer has a balance-of-payments problem. Similar conclusions have been reached by Depres-Kindleberger-Salant on the grounds that the U.S. plays the role of the world banker. Discuss.
    3. Discuss the implications of fixed and flexible exchange rates for national monetary and fiscal policies under the assumption of perfect capital mobility.

Source: Johns Hopkins University. Eisenhower Library, Ferdinand Hamburger, Jr. Archives. Department of Political Economy Series 6. Box 3; Folder: “Graduate Exams, 1933-1965”.

Image Source: Portrait of Bela Balassa in the Johns Hopkins University Yearbook, Hullabaloo 1976.

 

Categories
Bibliography Johns Hopkins M.I.T. Public Finance

M.I.T. History of Public Finance Bibliography. Dewey, 1936

As a graduate student at M.I.T. nearly fifty years ago, I spent a good part of my time when on campus at one of the many study desks along the windows at Dewey Library. At that time I had no idea who Davis Rich Dewey (after whom the library had been named) was. I presume this was true for most of my classmates too, M.I.T. not being known for  the study of the history of economics, though Paul Samuelson’s continuous interest in casting old theories in mathematical form was by no means chopped liver. 

As is noted in the short biography below, Dewey’s long career neatly overlapped with the first half-century of economics as a distinct academic discipline in North American universities. Thus it is fitting that Economics in the Rear-View Mirror gather and preserve artifacts left by Dewey in the course of his research and teaching.

Dewey’s magnum opus Financial History of the United States, first published in 1903, went through twelve editions (seven revisions) by 1934He dedicated the book to the Seminary in History, Politics, and Economics at Johns Hopkins University which he attended from 1883 to 1886. That dedication immediately follows the brief biography. This in turn is followed by a fully-linked fourteen item bibliography of general works on the history of U.S. public finance suggested “for students, teachers, and readers.”. Further suggestions by Dewey will be added sometime sooner or later, so stay tuned.

_____________________________

Davis Rich Dewey, 1858-1942

Davis Rich Dewey was Professor of Economics at M.I.T. and one of several people who helped shape the profession of economics as it is practiced today. Best known for his writings on United states economic history, his professional career spans fifty years (1886-1940), the formative period of the modern economics profession.

In 1883 Davis R. Dewey entered the graduate department of economics at Johns Hopkins University, secured a fellowship, and spent summers working as a correspondent for Bradstreet’s Financial Review. He graduated from Johns Hopkins with the doctorate in 1886 having studied history, economics and political economy. His Ph.D. thesis, entitled “A History of American Economic Literature…” was a survey of the practice of the early U.S. economics profession.

Upon his graduation, Dewey received an appointment as instructor in history and political science at the Massachusetts Institute of Technology. From the first he was integrally involved in research, publishing his first articles, “Municipal Revenue from Street Railways” [AEA Publications, Vol. II, No. 6 (1888), pp. 551-562] and “A Syllabus on Political History since 1815…” in 1889.

At M.I.T. Dewey served as an Instructor (1886-1888), then Assistant Professor of Economics and Statistics (1888-1889), Associate Professor (1889-1892) and finally Professor and Department Chairman (1893-1933). He taught a course in engineering administration from 1913- 1931, when a separate department of engineering administration was created, largely due to his efforts. He served as the Chairman of the M.I.T. Faculty from 1911-1913.

Dewey was influential in the internal affairs of two major professional organizations, the American Economic Association and the American Statistical Association. While still a graduate student, he had participated in the founding meeting of the American Economic Association and in 1909 he became its president. When that Association’s journal The American Economic Review was started in 1911, he served as its first editor, a post he held until 1940. The medal on the Dewey Library homepage was awarded to Davis R. Dewey upon the occasion of his retirement as editor of American Economic Review in 1940. Also in his first year of service at M.I.T., he became a member and was elected secretary of the American Statistical Association, an office he held until 1906. As secretary, and as a member of the Publications Committee, Dewey helped to edit the publications of that organization as well.

Davis R. Dewey was interested in the quality of education, as demonstrated by the following quotation,

“The Student will too often leave with…no systematic knowledge of the economic world, nor any well-defined theory of its workings. There must therefore be a far greater insistence upon…methods which will improve the missing experience.”

Davis R. Dewey was an associate of M.I.T. President Francis Amasa Walker whose Discussions in Economics and Statistics [Volume I: Finance and Taxation, Money and Bimetallism, Economic Theory. Volume II Statistics, National Growth, Social Economics] he edited for publication in 1899, shortly after Walker’s death. He was also associated with the editor Albert Bushnell Hart. Davis R. Dewey wrote his acclaimed Financial History of the United States for Hart’s American Citizen Series in 1903, and a volume entitled National Problems for Hart’s American Nation Series in 1907. In 1904 Financial History of the United States won the John Marshall Prize offered by Johns Hopkins. Dewey was a contributor to Palgrave’s Dictionary of Political Economy, the New International Encyclopedia Americana, Encyclopedia Britannica, American Year Book and the Commonwealth History of Massachusetts.

A representative of the modern field of economics, Davis R. Dewey was indifferent to theorizing which had little to do with empirical fact. He was above all a practitioner, insisting that applied knowledge we the true realm of the academic economist. Davis R. Dewey also maintained a lively interest in the politics of academe and followed several academic freedom cases of his day.

He died on December 13, 1942. The Dewey Library was named in his memory.

Written by Keith Morgan, Dewey Library Economics Bibliographer, 1994

Source: Webpage “Davis Rich Dewey, 1858-1942,” MIT Libraries, Dewey Library for Social Sciences and Management. Links added by Economics in the Rear-View Mirror.

_____________________________

Financial History of the United States (12th edition).
New York: Longmans, Green and Co., 1936.
by
Davis Rich Dewey, Ph.D., LL.D.,
Emeritus Professor of Economics and Statistics
Massachusetts Institute of Technology

To the Seminary
of the
Department of History, Politics, and Economics
of Johns Hopkins University,

Of which the author was a member from 1883 to 1886. Under the guidance of Adams, Ely, and Jameson, we read and learned. The first has gone, leaving affectionate memories and organized activities of permanent usefulness; the others are still doing their work in a spirit of broad-minded sympathy and fine scholarship.

_____________________________

Suggestions for Students, Teachers, and Readers

[Following three pages dedicated to general works on U.S. political history and biography, Dewey offers almost seventeen full pages dedicated to the subject of public finance. In this post we begin with the transcription of the most general works in public finance Dewey recommends. The curator of Economics in the Rear-View Mirror has been able to find links to the fourteen annotated items in Dewey’s list.]

II. Financial Histories

There are but few histories devoted exclusively to public finance; only one, indeed, that by Bolles, covers the general field over an extended period. The reader must therefore rely upon works on taxation, the tariff, coinage, and banking, and for special topics and episodes will often find the most satisfactory treatment in the political histories and biographies already referred to. The following volumes represent those which are most general in their treatment; of these the works of Bolles and Noyes are especially to be recommended; the narrative by Bolles stops with 1885, while the smaller work by Noyes is confined to the period 1865-1907. There is a great need of detailed works on the expenditures of the government and the various phases of treasury administration.

Adams, Ephraim D. The Control of the Purse in the United States Government. (Reprinted from the Kansas University Quarterly, April, 1894.) — An academic study of the debates in Congress on the interpretation of constitutional provisions relating to treasury management, loans, taxation, and money bills. Careful references are given.

Bolles, Albert Sidney. American Finance, with Chapters on Money and Banking. (N. Y., 1901.) — Especially valuable on expenditures; treats also of State finance. A discussion of present conditions rather than historical.

Bolles, Albert Sidney. The Financial History of the United States. (2d ed. N. Y., 1884-1886. 3 vols.) — Vol. I includes the period 1774-1789; II, 1789-1860; III, 1861-1885. The only single work which covers an extensive period; it represents research, and is closely restricted to questions of finance; no attempt is made to sketch in the political and social background, and the reader may be confused without preliminary reading. The author leans to protection, and takes the banker’s point of view in questions of currency. The work is especially valuable for chapters on accounting and expenditures. Referred to as Bolles.

1st edition (1879), Vol. I (Frank Taussig’s copy!)
1st edition (1883), Vol. II
1st edition (1886), Vol. III
2nd edition (1884), Vol. I
2nd edition (1885), Vol. II
2nd edition (1885), Vol. III
3rd edition (1892), Vol. I

Bourne, Edward Gaylord. The History of the Surplus Revenue of 1837. (N. Y., etc., 1885.) — A brief, scholarly monograph with abundant references and bibliography. In addition to the historical account, it summarizes the earlier proposals of distribution of surplus funds in the treasury.

Bronson, Henry. Historical Account of Connecticut Currency, Continental Money, and the Finances of the Revolution. (In New Haven Colony Hist. Soc. Papers, Vol. I. New Haven, 1865.) — This is more than a local study; the author is drawn into a general review of the financial measures of the Revolutionary War. The essay is scholarly and the style vigorous.

Bullock, Charles Jesse. Finances of the United States, 1775-1789, with Especial Reference to the Budget. (Madison, 1895. Univ. of Wisconsin. Bulletin, Economics, etc., Vol. I, No. 2.)  — The best monograph on the finances of the Revolutionary period, with bibliographies at the beginning of each chapter. Indispensable to the advanced student.

Kearny, John Watts. Sketch of American Finances, 1789-1835. (N. Y., 1887.) — A brief study of 150 pages, clear and helpful in questions concerning the treatment of the debt. Little attention is given to taxation.

Noyes, Alexander Dana. Thirty Years of American Finance, 1865-1896. (N. Y., etc., 1898.) — Treats the earlier period very briefly, but is of special value for 1878-1895. Relation of public finance to the money market is given prominence. This has been replaced by Forty Years of American Finance (1909), bringing the history down to 1907. The references to the earlier edition have been allowed to stand.

Schuckers, Jacob William. Brief Account of the Finances and Paper Money of the Revolutionary War. (Philadelphia, 1874.) — The style is somewhat rhetorical, and, while the writer has on the whole chosen sound authorities, the essay does not indicate a very wide research. Is an interesting account within a moderate space.

Scott, William A. The Repudiation of State Debts. (N. Y. etc., 1893.) — Chapters 2-6 are historical, describing various acts of repudiation in twelve States. Of value as explaining some of the remote influences affecting federal credit, 1825-1850.

Spaulding, Elbridge Gerry. History of the Legal Tender Paper Money issued during the Great Rebellion. (Buffalo, 1869.) — The title is hardly accurate; the volume is largely a collection of documents, speeches, etc., relating to the legal tender acts.

Sumner, William Graham. The Financier and Finances of the American Revolution. (N. Y., 1891. 2 vols.) — Contains a mine of valuable material, but is not clearly arranged.

Vol. 1 (1892)
Vol. 2 (1892)

Sumner, William Graham. A History of American Currency. (N. Y., 1874.) — A series of topical notes designed for reference rather than consecutive reading.

Wells, David Ames. Practical Economics. (N. Y., etc., 1885.) — Treats of the silver question, tariff revision, and, most valuable of all, experience of the United States in taxing distilled spirits, subsequent to the Civil War.

 

Source: Davis Rich Dewey, Financial History of the United States (Twelfth edition). New York: Longmans, Green and Co., 1936, pp. xi-xiii.

Image Source: Davis Rich Dewey portrait at the MIT Museum website. Retouched and colorized by Economics in the Rear-View Mirror.

Categories
Exam Questions Johns Hopkins Suggested Reading Syllabus Undergraduate

John Hopkins. Economic Fluctuations and Fiscal Policy. Course outline, reading list, exams. Domar, 1956

Evsey Domar turned 42 years old towards the end of the Spring term of 1955-56 when he taught his intermediate fiscal policy course to Johns Hopkins’ undergraduates. From his papers at Duke’s Economists’ Papers Archive we can bring together the tightly focussed reading list, two midterm exams, and the final exam for Political Economy 4. 

One notes that the actual dates of the mid-term exams were lagged one week relative to the announced dates in the syllabus. Happens to the best of us. I wonder if students still (ever?) read the syllabus back in the middle of the 20th century. 

________________________

Course Announcement

Political Economy
Specialized intermediate work

Economic Fluctuations and Fiscal Policy 4. Professor Domar. Three hours weekly, second term.

The nature and causes of economic fluctuations. The economic role of government. Principal policy measures designed to achieve economic stability.

Prerequisite: Political Economy 3, or its equivalent.

Source: Johns Hopkins University. Undergraduate Programs, Announcements of Courses 1955-1956 in Circular 1955-1956. New Series 1955, Number 8, p. 102.

________________________

Course Outline and Readings

THE JOHNS HOPKINS UNIVERSITY
ECONOMIC FLUCTUATIONS
AND FISCAL POLICY

(Political Economy 4)

E. D. Domar
Spring Term 1955-56

Course Schedule

SOURCES:

On College Reserve:

Colm, Gerhard, Essays in Public Finance and Fiscal Policy, Oxford University Press, New York 1955.

Due, John F., Government Finance—an Economic Analysis, Richard D. Irwin, Inc., Homewood, Ill., 1954.

Gordon, Robert A., Business Fluctuations, Harper & Brothers, New York, 1952.

Lindholm, Richard W., J. J. Balles, J. M. Hunter, Principles of Money and Banking Related to National Income and Fiscal Policy,W. W. Norton & Co., New York, 1954.

Public Finance and Full Employment, published by the Board of Governors of the Federal Reserve System, Washington, 1945.

Ritter, Lawrence S., Money and Economic Activity, Houghton Mifflin Co., Boston, 1952.

To Be Acquired by the Students:

Maxwell, James A., Fiscal Policy, Henry Holt & Co., New York, 1955.

Economic Report of the President, January 1956.

SCHEDULE:

Week of February 13th:

Maxwell, Ch. 1,
Ritter, pp. 20-36
Lindholm, pp. 17-31.

Week of February 20th:

Ritter, pp. 99-113,
Maxwell, Ch. 2.

Week of February 27th:

Ritter, pp. 120-130,
Lindholm, pp. 330-348

Week of March 5th:

Lindholm, pp. 370-408.

HOUR EXAMINATION: March 12th

Week of March 12th:

Maxwell, Ch. 3, 4 & 5.

Week of March 19th:

Maxwell, Ch. 6, 7, & 8,
Federal Reserve, pp. 1-21,
Colm, pp. 188-219.

Week of March 26th:

Maxwell, Ch. 9, 10, & 11,
Colm, pp. 258-286.

Week of April 2nd:

Maxwell, Ch. 12 & 13,
Federal Reserve, pp. 22-52,
Review – Due, pp. 29-61, 427-39.

Week of April 9th:

Maxwell, Ch. 14 & 15.

HOUR EXAMINATION: April 16th

Week of April 16th:

Federal Reserve, pp. 53-68, 101-130.

Week of April 23rd:

Review – Gordon, Ch. 13 & 14, and pp. 559-74.

Week of April 30th:

Gordon, Ch. 16, 17 & 18.

Week of May 7th:

Economic Report of the President

Week of May 14th:

Economic Report of the President

Week of 21st:

General Review of the Course

Source: Duke University. David M. Rubenstein Rare Book and Manuscript Library. Economists’ Papers Archive. Papers of Evsey Domar. Box 15, Folder “MacroEconomics, Old Reading Lists”.

________________________

First Hour Test

THE JOHNS HOPKINS UNIVERSITY
Economic Fluctuations and Fiscal Policy
(Political Economy 4)
Spring Term 1955-56

March 19, 1956

E.D. Domar

Answer all questions in any order you wish. Indicate carefully every step in your reasoning.

  1. (40%) Write a comprehensive essay on the subject of “Central Bank Monetary Policy” with special reference to our Federal Reserve System. Your essay should include the following points:
    1. The structure of the Federal Reserve System.
    2. The relation between commercial and Federal Reserve Banks.
    3. Objectives of Federal Reserve Policy.
    4. Powers given to the Federal Reserve System and methods used by it to achieve the objectives indicated in (3) under different economic conditions.
      1. General measures
      2. Selective measures
    5. Evaluate the performance of the Federal Reserve System since its inception.
      How successful has it been in achieving the objectives stated in (3)?
    6. Conclusion: the virtues and defects of Monetary Policy.
  2. (25%) Indicate clearly how DEMAND DEPOSITS, REQUIRED RESERVES, EXISTING RESERVES and EXCESS RESERVES of the commercial member banks taken as a whole are affected by the following transactions.
    Assume that all payments are made by check, that the member banks add all receipts to, and subtract all amounts paid out from, their reserves with the Federal Reserve Banks, and that the U.S. Treasury keeps all its funds with the Federal Reserve Banks:
  3. 25% Legal requirements are 15 per cent.
  4. When a transaction consists of several parts, indicate each part separately and then show the total effect.
    AFTER EACH TRANSACTION GIVE A BRIEF VERBAL ANALYSIS OF ITS ECONOMIC EFFECTS.

    1. The U.S. Treasury collects $15 million of corporate income taxes from the U.S. Steel Corporation and uses the proceeds to redeem a bond held by Mr. Smith who deposits the check with his bank.
    2. Same as (1), but the bond is held by the First National Bank.
    3. Jones borrows $1000 from the First National Bank. After a while he uses the proceeds to meet his payroll. His employees invest their earnings in Federal bonds.
    4. The U.S. Treasury sells bonds for $100 million to the public, and uses the proceeds to buy land for highway construction. The owners of the land deposit their checks at their banks. The Federal Reserve Banks buy $100 million worth of Federal bonds from (a) the public, and (b) commercial banks.
    5. The Federal Reserve Board changes reserve requirements from 20 to 18 per cent. (Assume that the amount of deposits outstanding equals to $100 billion.) Thereupon banks extend loans to their customers of $1 billion.
  5. (35%) Write a comprehensive essay on the subject of “The Identity and Divergence between Private and Social Cost.” Illustrate your discussion with examples. Why is this question important to the subject matter of our course and to economic policy in general. (No credit will be given for vague generalities.)

Source: Duke University. David M. Rubenstein Rare Book and Manuscript Library. Economists’ Papers Archive. Papers of Evsey Domar, Box 16, Folder “Misc. Examinations”.

________________________

Second Hour Exam

THE JOHNS HOPKINS UNIVERSITY
ECONOMIC FLUCTUATIONS
AND FISCAL POLICY

(Political Economy 4)
Spring Term 1955-56

Hour Examination
April 23, 1956

E.D. Domar

Answer all questions in any order you wish. Indicate carefully every step in your reasoning. No credit will be given for vague generalities.

  1. (15%) Define and describe the following terms or expressions and indicate their use in economic discussions:
    1. The Multiplier;
    2. Parity;
    3. Balanced budget theorem;
    4. Cash vs. conventional budget;
    5. Carryovers and carrybacks;
    6. Income elasticity of taxation;
    7. Regressive taxation;
    8. Payroll taxes;
    9. Grants-in-aid;
    10. Accelerated depreciation.
  1. (20%) Write a comprehensive essay on the subject of “Built-in Flexibility as an Instrument of Fiscal Policy.” Explain what is meant by this expression, how this instrument works, how effective it is likely to be, and what can be done to increase its effectiveness. Give a critical evaluation. Be as comprehensive and specific as you can.
  2. (20%) Write a comprehensive essay on the subject of “The Agricultural Problem in the United States since the Second World War.” Explain the origin and causes of the problem, government policies which have been adopted, and their effectiveness in dealing with the problem. Indicate and justify your own recommendations.
  3. (30%) Analyze with great care all important economic effects of agricultural price support program on the assumption of (1) that the funds for this purpose are raised by borrowing, and (2) that they are raised by taxation, in both cases under conditions of (a) unemployment, and (b) full employment. Indicate in all cases what kind of borrowing and what kind of taxation you have in mind. Give examples. When would you recommend one or the other method?
  4. (15%) “The main objective of the Federal policy should be not the balancing of the Federal budget, but of the national economic budget.” Comment.

Source: Duke University. David M. Rubenstein Rare Book and Manuscript Library. Economists’ Papers Archive. Papers of Evsey Domar, Box 16, Folder “Misc. Examinations”.

________________________

Final Exam

THE JOHNS HOPKINS UNIVERSITY
ECONOMIC FLUCTUATIONS
AND FISCAL POLICY

(Political Economy 4)

FINAL EXAMINATION – Three hours
June 1, 1956

E. D. Domar

Answer all questions. Be specific.

  1. (25%) Compare and contrast monetary and fiscal policies as methods of achieving economic stabilization (reasonably full employment without inflation) in a growing society. Include (but don’t limit yourself to) the following points:
    1. The theoretical foundation of each;
    2. Methods used by each;
    3. Effects on distribution of income and wealth;
    4. Social and political effects;
    5. Their effectiveness and limitations.

Do they overlap? Can you work out a synthesis of both?

  1. (10%) Describe how business fluctuations spread internationally and discuss critically the various measures for insuring international stability that have been suggested.
  2. (15%) Suppose that sizable gold deposits were discovered in this country (a) in 1933 and (b) in 1955. Trace the economic effects of the mining of this gold as completely as you can, both on the American economy and on that of other countries.
  3. (20%) Describe the origin, functions and performance of the Council of Economic Advisers from its beginning.
    State and evaluate the basic economic philosophy and the major recommendations of the 1956 Economic Report of the President.
  4. (15%) “One of the first objectives of this Administration should be at least a partial repayment of the Federal Debt. To do otherwise is to undermine the integrity on which this Administration is founded, and to adopt a course which inevitably loads to higher taxes, inflation, the destruction of our national wealth and economic insolvency.” Comment fully.
  5. (15%) Discuss SAVING as an economic problem.

Source: Duke University. David M. Rubenstein Rare Book and Manuscript Library. Economists’ Papers Archive. Papers of Evsey Domar, Box 16, Folder “Misc. Examinations”.

Image source: Duke University. David M. Rubenstein Rare Book and Manuscript Library. Economists’ Papers Archive. Papers of Evsey Domar, Box 18, Folder “Photographs Domar”. Copy also available at the MIT Museum website. Colorized by Economics in the Rear-view Mirror.

Categories
Berkeley Columbia Cornell Economists Harvard Illinois Johns Hopkins Wisconsin

USA. Joseph Schumpeter’s Roadshow. 1st Quarter, 1914

Joseph Schumpeter spent the 1913-14 academic year as Austria’s first exchange professor at Columbia University. But before heading home, he went on a whirlwind tour of American universities as documented in the following collection of news reports. Cornell, George Washington, Johns Hopkins, the Universities of Illinois, Wisconsin, and California, and apparently culminating with lectures in Taussig’s Ec 11 course at Harvard. All this between mid-January and mid-March 1914.

I have not seen the above portrait of Schumpeter before. He looks much less like Nosferatu’s twin and one could say has even leading-man material if only his ears were pinned back a notch.

_____________________________

Dr. Schumpeter Near End of Course as Austrian Exchange Professor at Columbia University

Professor Joseph A. Schumpeter, who was sent to this country by the Austrian Government as an exchange professor, will soon complete his course of lectures on economic theory and on the problem of social classes, at Columbia University, where he has been since last October, and will visit a number of other leading universities in this country.

Professor Schumpeter was born in 1883, in Triesch in the Austrian Province of Moravia, and was educated at the “Theresianum” in Vienna. Then he entered the University of Vienna where he took his degree of Doctor in Law and Political Science in 1906, and gained locally some representation within the little circle of students of economic theory called the Austrian School. After spending some years in travel, he began lecturing on economies at the same university from which he was, at the age of 26, called to the chair of Political Economy in the University of Gernowitz. In 1911 he accepted a call to the University of Graz in Styria. When the Austrian Government, following the example given by the German Government, concluded an agreement with Columbia University for the exchange of professors, Schumpeter was selected to be the first visiting professor in this country.

Source: The Brooklyn Daily Eagle. January 14, 1914, p. 12.

_____________________________

Predicts More War in Balkan Frontier

“Conditions as they exist in the Balkans now cannot last, I am sorry to say that the sad story of crime and suffering that we have been witnessing we shall have to see over again before long,” said Prof. Joseph Schumpeter in his lecture last night on “Austria’s Balkan Policy.”

“The Balkan situation awakens in us a multitude of passions,” said Dr. Schumpeter. “We see burning and murdered villages, and conditions growing worse and worse. Austria is very little known.

“A lot of false notions have arisen concerning Austria. It is a country of 50,000,000 inhabitants composed of a combination of different races and therefore gives statesmanship tasks of a peculiar kind. It is impossible to appeal to national patriotism in Austria for it is composed of several races apathetic to each other. The majority of the people are Slavs, but there are a great many Germans, Romanians, Italians and Servians. It is very difficult to adjust their claims for national supremacy.

“To keep the Turkish frontier is still Austria’s main care.

“What Austria wanted, and wants still, is to have a group of states on national lines so arranged that they will last for some time and not be under the influence of Russia. The Albanians held their own against Turkey for some time and finally Austria made a treaty with Italy that, no matter what happened to Turkey, they should combine to save the Albanian state.”

Source: The Ithaca Journal. January 17, 1915, p. 5.

_____________________________

Admires Quick Wit Found in America

Professor Joseph Schumpeter of the University of Graz, Austria, expresses himself as delighted with Cornell University and everything he has seen here. Professor Schumpeter, who gave the lecture on “The Balkan Policies of Austria,” is now Exchange Professor at Columbia University. He has gained the reputation of being one of the most promising economists of Europe.

“You have a wonderful University here, splendidly equipped,” he declared. “The situation is ideal. I have been very much interested in my work at Columbia and feel that we Europeans can learn a lot from you. My work at Columbia has been mostly with the graduate students and I have not been able to get into as close touch with the undergraduates as I would have liked to. I have been especially struck by the quick-wittedness and energy of the American undergraduates. They also have an aptitude for intelligent discussion which is lacking in Europe. The whole spirit of fellowship is so splendid.

“The social life in America is remarkably pleasant. In other countries you take much longer to make friends. In my short stay here I have already made scores of excellent friends. American audiences are also so pleasant to talk to. I feel less intellectual sympathy while talking before a German audience than I do here. Unlike Europe all classes of society seem equally interested, the workingmen as much so as the richest families.”

Source: The Ithaca Journal, Jan 19, 1914, p. 9.

_____________________________

Talk on the Balkans by Prof. Schumpeter
University of Gratz Educator Entertains Audience at National Museum

Prof. Joseph Schumpeter delivered a lecture on the present and prospective situation in the Balkans at the New National Museum last night, to which the general public was invited, the audience including students of the eastern question and a number of diplomats. Dr. Schumpeter is exchange professor between the University of Gratz, where he is professor of political science and economics, and Columbia University, New York, where he has just completed his series of lectures.

Last night’s address was delivered under the auspices of George Washington University, and Dr. Schumpeter was introduced by Rear Admiral Charles Herbert Stockton, U. S.N., retired, president of George Washington.

Dr. Schumpeter gave an historical account of the development of the Ottoman empire from the conquest of Constantinople in 1453 up to the present time. He declared that outside of Greece, where the situation has been practically clarified, the Balkan troubles have not been set at rest, and that further trouble may be expected in the Balkan countries at any time.

Real Root of Troubles.

He pointed out that the real root of many of the troubles of those countries has been differences because of race, the clash between Mohammedan and Christian, which he said is likely to continue to the end of the world. He offered, for example, the experiences of Great Britain, in charge of the largest number of Mohammedans in the world, in India.

Dr. Schumpeter devoted considerable attention to the part that Austria-Hungary is playing in the Balkan situation. Since 1908 the Balkan provinces of Bosnia and Herzegovina have been recognized as a part of Austria, and it is around the possession of these provinces that considerable interest lies. Austria-Hungary was permitted by the congress of Berlin in 1878 to occupy and administer these two Balkan provinces, but it was not until 1908 that they were formally annexed.

Source: Evening Star (Washington, D.C.), February 5, 1914, p. 9.

_____________________________

Balkan War Policy of Austria Defended
Prof. Schumpeter Says Nation Must Control of Principalities Along the Danube.
Great Britain is Criticised.

John Bull was said to have in his charge the greatest Mohammedan power in the world by Prof. Joseph Schumpeter, professor of political science and economics at the University of Gratz, Austria, in an Interesting lecture last night at the National Museum.

“No less than 90,000,000 Mohammedans are under British rule, yet England has seen fit to attack the right of Austria to establish a mere legal pact by taking over Herzegovina and Boris and assuring to these two countries safety and security,” said Prof. Schumpeter.

Rear Admiral Stockton, president of Georgetown University, under the auspices of which institution the lecture was given, presided.

Prof. Schumpeter gave a comprehensive historical account of the development of the Ottoman empire since the conquest of Constantinople, in 1453, up to the present day.

In closing, Prof. Schumpeter asserted that in his opinion, outside of Greece, where he said the situation has been pretty well clarified, the Balkan troubles have not been definitely put at rest. He made it clear that he looked for further trouble.

Source: The Washington Herald, February 5, 1914, p. 3.

_____________________________

Predicts Third Balkan War.
Prof. Schumpeter of Austria, Is Pessimistic in Lecture.

Under the auspices of George Washington University, Prof. Joseph Schumpeter, dean of political science and economics at the University of Gratz, Austria, and exchange professor between his university and Columbia, spoke Wednesday in the auditorium of the new National Museum on “The Balkan Situation from the Austrian Viewpoint.” Among those present were Konstantin von Masirevich, first secretary, and the Baron Freudenthal, attache of the Austrian embassy; Rear Admiral C. H. Stockton, president of George Washington, Prof. Richard Cobb, secretary of the university; Dean Charles E. Munroe, Dean Charles Noble Gregory and L. Cleveland McNemar, assistant professor of international law.

Prof. Schumpeter claimed that the Austrian annexation of Bosnia and Herzogovina in 1910 was warranted by circumstances. He said that Balkan peace is merely transitory; that another war is sure to come.

Source: The Washington Post, February 8, 1914, p. 2.

_____________________________

Johns Hopkins University.

Two foreign educators lectured at the Johns Hopkins university last week. Dr. Joseph Schumpeter, of the University of Gratz, and first Austrian exchange professor at Columbia, delivered five lectures before the department of political economy….

Source: The Oregon Daily Journal. February 15, 1914, p. 48.

_____________________________

Vienna Man at Madison.
Exchange Professor Studies Wisconsin Industrial Laws.

(Special to The Northwestern.) Madison, Wis., Feb. 10. — Dr. J. A. Schumpeter, exchange professor from Vienna to Columbia university, spent today here investigating the work of the state industrial commission. He will leave tonight for St. Paul, where he intends to make a similar investigation. Dr. Schumpeter is a recognized expert on labor legislation, and in addition to investigating the work of the commission held a conference with Prof. John R. Commons.

Source: The Oshkosh Northwestern, February 10, 1914, p. 9.

_____________________________

Professor Schumpeter has very busy two days here.
Austrian makes four addresses and attends several affairs.

Four addresses were given by Prof. Josef Schumpeter during his stay here from last Saturday morning until Sunday night, when he left for the University of California via Chicago. All of his addresses were along the line of the social sciences in which he enjoys wide fame for his great ability.

Professor Schumpeter’s first talk was given Saturday noon to the University Club where he took lunch. There he gave a talk on smoke. His second, and most important address, was to the combined seminars of the social science departments. It was given in room 304 of Lincoln Hall to an audience which contained almost every faculty and student member of the two seminars. His discussion was upon the theory of economic development. The lecturer paid especial attention to the place of interest and economic crises in static and dynamic states. Although he is an Austrian. and from the University of Graz, his lecture was delivered in perfect English, and was of profound interest to those who were privileged to attend.

With some eighteen University people, Professor Schumpeter took dinner at the Beardsley Saturday evening. Here again he spoke, this time giving an address on “The Austrian Attitude to the Balkan Situation.” He was followed by Professors E. B. Greene, A. H. Lybyer, L. M. Larson and W. F. Dodd, each of whom spoke on some aspect of world politics.

A reception was given the Professor at the home of Dean Kinley on Sunday afternoon. Here he addressed those present on “The Aspects of Austrian Social and Political Life in University Government.” Sunday night, Professor Schumpeter took the train for Chicago.

Source: The Daily Illi (Urbana, Illinois). February 17, 1914. Page 4.

_____________________________

Talks of Marx’ Economy

University of California, Feb. 25. — Dr. J. A. Schumpeter, eminent Austrian economist, addressed an audience at California hall yesterday on “The Economy of Karl Marx.” Among the points he made were that one could believe in Marx’ doctrines without being a Socialist; his theories of value and exploitation of the working class were receiving more and more general acceptance; he was a “flaming propagandist whose followers regarded him as little less than inspired.”

He addressed also during the day classes in economics on interest rates and classical and modern economic theories.

Source: Oakland Tribune. February 25, 1914, p. 4.

_____________________________

Brings Austria’s Message to Both Columbia and Harvard

The first Austrian exchange professor at Columbia will deliver a series of lectures on economic theory at Harvard in March. His lectures will be in connection with some of the courses given by Professor Taussig on economic theory.

Professor Schumpeter was born in Triesch, Moravia, in 1883, and received his early education in Vienna, where he also attended the University. He was awarded the degree of doctor of law and political science in 1906 and after spending several years in travel, established himself as a docent at the University of Vienna in 1909. A few months later, he was appointed professor of political economy in the University of Czernowitz, and in 1911 he was appointed professor of political economy in the University of Graz.

Professor Schumpeter’s own system of economic theory is developed in two books, “Wesen und Hauptinhalt der Theoretischen Nationalökonomie” and “Theorie der Wirtschaftlichen Entwicklung.” In addition, he has published a number of important papers. He has also contributed a history of economic theory to Schönberg’s “Handbuch der Politischen Oekonomie,” which is to appear shortly, and has furthermore prepared a treatise on banking law for a manual of mercantile law to be published in the near future.

In recent years Professor Schumpeter’s interest has been largely in the field of sociology, but he has not yet published anything in this department. As an economist, Professor Schumpeter is a member of the Austrian school — the brilliant group of writers headed by Böhm-Bawerk and Wieser of the University of Vienna, who have rescued economic theory from the eclipse with which it was for a time threatened by the able but exaggerated criticisms of the leaders of the German historical school. Though agreeing with his Viennese colleagues in many of their theories, he has shown great independence and originality in his treatment of the phases of economics, such as the interest problem, to which he has given particular attention. Thus, in spite of his comparative youth, he has won a place in the very front rank of contemporary European economists. His fluent command of spoken English and his intimate knowledge of American economic literature make him a most attractive lecturer.

Source: Boston Evening Transcript. February 4, 1914.

_____________________________

“The Theory of Crises”at 4.30
March 16, 1914

Professor Josef Schumpeter, an Austrian economist of the University of Vienna, will lecture upon “The Theory of Crises” before the Seminary of Economics in Upper Dane this afternoon at 4.30 o’clock. Professor Schumpeter has written two books upon Economic. Theory which are of high quality, and have attracted a great deal of attention.

Source: The Harvard Crimson. March 16, 1914.

_____________________________

Noted Economist’s Last Lecture

All members of the University interested in economics are invited to hear Professor Josef Schumpeter, of the University of Vienna, lecture upon “Economic Theory” in Professor Taussig’s course, Economics 11, this afternoon at 2.30 o’clock. This will be the last of a series of lectures given by Professor Schumpeter, who is this year conducting courses at Columbia University. The lecture will take place in Emerson H.

Source: The Harvard Crimson. March 18, 1914.

_____________________________

Prof. Schumpeter Sails.
Says That America Made a Deep Impression on Him.

Professor Joseph Schumpeter, the first Austrian exchange professor in America, who is returning to his home University of Graz, Austria, sailed on the Martha Washington yesterday. The professor has lectured on social problems, money systems, democracy and other branches of science, has been with Columbia University until the end of January, when he left for a tour through all of the leading universities of this country, the tour extending to San Francisco. Mr. Schumpeter said that he was sorry to leave America, which had made a deep impression upon him.

“The big American universities,” the professor stated, “are far better than the average Austrian and even European university. America has a bigger and better body of scientists at each university, and the student’s material is of a much higher type than that of European schools. The American student wants to learn. He has the earnest desire to go to the bottom of science. He wants to make headway in the world, whereas the Austrian student visits a university for reasons of tradition, social standing and title.”

Asked what he thought the greatest American achievement, the professor answered that the “one-man management” was most appealing to him. It was far better, he claimed to have one man run a business, a university, and even a political party, than to have the European system of sharing power and responsibilities.

Four pretty young sisters, all of whom are ardent suffragists, left on board the big Austrian liner for Italy, France and Germany. The fair travelers are Misses Catherine, Ella S., Grace and Margaret Switzer of Manhattan. Their purpose is to show their European sisters how superfluous man really is, for never during their trip will they tolerate or accept the services of any man nor will they speak to any man or stand for being addressed by a man.

Source: The Brooklyn Daily Eagle. March 22, 1914, p.74

Image Source: Boston Evening Transcript. February 4, 1914. Colorized by Economics in the Rear-view Mirror.